You are on page 1of 118

www.allsyllabus.

com

Many electric circuits are complex, but it is an engineer’s goal to reduce their complexity to
analyze them easily. In the previous chapters, we have mastered the ability to solve networks
containing independent and dependent sources making use of either mesh or nodal analysis. In
this chapter, we will introduce new techniques to strengthen our armoury to solve complicated
www.allsyllabus.com
networks. Also, these new techniques in many cases do provide insight into the circuit’s operation
that cannot be obtained from mesh or nodal analysis. Most often, we are interested only in the
detailed performance of an isolated portion of a complex circuit. If we can model the remainder
of the circuit with a simple equivalent network, then our task of analysis gets greatly reduced and
simplified. For example, the function of many circuits is to deliver maximum power to load such
as an audio speaker in a stereo system. Here, we develop the required relationship betweeen a
load resistor and a fixed series resistor which can represent the remaining portion of the circuit.
Two of the theorems that we present in this chapter will permit us to do just that.

3.1 Superposition theorem

The principle of superposition is applicable only for linear systems. The concept of superposition
can be explained mathematically by the following response and excitation principle :
i1 ! v1

i2 ! v2

then; i1 + i2 ! v1 + v2
The quantity to the left of the arrow indicates the excitation and to the right, the system
response. Thus, we can state that a device, if excited by a current i1 will produce a response
v1 . Similarly, an excitation i2 will cause a response v2 . Then if we use an excitation i1 + i2 , we
will find a response v1 + v2 .
The principle of superposition has the ability to reduce a complicated problem to several easier
problems each containing only a single independent source.

vtu.allsyllabus.com
www.allsyllabus.com

160 j Network Theory

Superposition theorem states that,


In any linear circuit containing multiple independent sources, the current or voltage at any
point in the network may be calculated as algebraic sum of the individual contributions of each
source acting alone.
When determining the contribution due to a particular independent source, we disable all
the remaining independent sources. That is, all the remaining voltage sources are made zero by
replacing them with short circuits, and all remaining current sources are made zero by replacing
them with open circuits. Also, it is important to note that if a dependent source is present, it must
remain active (unaltered) during the process of superposition.
Action Plan:

(i) In a circuit comprising of many independent sources, only one source is allowed to be active
in the circuit, the rest are deactivated (turned off).
(ii) To deactivate a voltage source, replace it with a short circuit, and to deactivate a current
source, replace it with an open circuit.
(iii) The response obtained by applying each source, one at a time, are then added algebraically
www.allsyllabus.com
to obtain a solution.
Limitations: Superposition is a fundamental property of linear equations and, therefore, can be
applied to any effect that is linearly related to the cause. That is, we want to point out that,
superposition principle applies only to the current and voltage in a linear circuit but it cannot be
used to determine power because power is a non-linear function.

EXAMPLE 3.1
Find the current in the 6 Ω resistor using the principle of superposition for the circuit of Fig. 3.1.

Figure 3.1

SOLUTION
As a first step, set the current source to zero. That is, the current source appears as an open circuit
as shown in Fig. 3.2.
6 6
i1 = = A
3+6 9

vtu.allsyllabus.com
www.allsyllabus.com

Circuit Theorems j 161

As a next step, set the voltage to zero by replacing it with a short circuit as shown in Fig. 3.3.

i2 =
2 3 6
= A
3+6 9

Figure 3.2 Figure 3.3

The total current i is then the sum of i1 and i2


12
i = i1 + i2 = A
9
EXAMPLE 3.2
www.allsyllabus.com
Find io in the network shown in Fig. 3.4 using superposition.

Figure 3.4

SOLUTION
As a first step, set the current source to zero. That is, the current source appears as an open circuit
as shown in Fig. 3.5.

Figure 3.5

vtu.allsyllabus.com
www.allsyllabus.com

162 j Network Theory

6

0
io = = 0:3 mA
(8 + 12) 103
As a second step, set the voltage source to zero. This means the voltage source in Fig. 3.4 is
replaced by a short circuit as shown in Figs. 3.6 and 3.6(a). Using current division principle,
iR2
iA =
R1 + R2

where
www.allsyllabus.com
jj
R1 = (12 kΩ 12 kΩ) + 12 kΩ
= 6 kΩ + 12 kΩ
= 18 kΩ
and R2 = 12 kΩ

) 
4 10 3 12 103  
iA =
(12 + 18) 103 
= 1:6 mA Figure 3.6
Again applying the current division principle,
00
io =
iA 12= 0:8 mA
12 + 12
Thus; io = io 0 + io 00 = 0:3 + 0:8 = 0:5 mA

Figure 3.6(a)

vtu.allsyllabus.com
www.allsyllabus.com

Circuit Theorems j 163

EXAMPLE 3.3
Use superposition to find io in the circuit shown in Fig. 3.7.

Figure 3.7

SOLUTION

www.allsyllabus.com
As a first step, keep only the 12 V source active and rest of the sources are deactivated. That is,
2 mA current source is opened and 6 V voltage source is shorted as shown in Fig. 3.8.

12

0
io =
(2 + 2) 103
= 3 mA

Figure 3.8

As a second step, keep only 6 V source active. Deactivate rest of the sources, resulting in a
circuit diagram as shown in Fig. 3.9.

vtu.allsyllabus.com
www.allsyllabus.com

164 j Network Theory

Applying KVL clockwise to the upper loop, we get

2  10 3
io
00
2  10 3
io
00
6=0
) io
00
=
4 
6
103
= 1:5 mA

Figure 3.9

www.allsyllabus.com
As a final step, deactivate all the independent voltage sources and keep only 2 mA current
source active as shown in Fig. 3.10.

Figure 3.10

Current of 2 mA splits equally.


000
Hence; io = 1mA

Applying the superposition principle, we find that

io = io 0 + io 00 + io 000
=3 1:5 + 1
= 2:5 mA

vtu.allsyllabus.com
www.allsyllabus.com

Circuit Theorems j 165

EXAMPLE 3.4
Find the current i for the circuit of Fig. 3.11.

Figure 3.11

SOLUTION

We need to find the current i due to the two independent sources.


As a first step in the analysis, we will find the current resulting from the independent voltage

www.allsyllabus.com
source. The current source is deactivated and we have the circuit as shown as Fig. 3.12.
Applying KVL clockwise around loop shown in Fig. 3.12, we find that

5i1 + 3i1 24 = 0
) i1 =
24
8
= 3A

As a second step, we set the voltage source to zero and determine the current i2 due to the
current source. For this condition, refer to Fig. 3.13 for analysis.

Figure 3.12 Figure 3.13

Applying KCL at node 1, we get


v1 3i2
i2 +7= (3.1)
2
v1 0
Noting that i2 =
3
we get, v1 = 3i2 (3.2)

vtu.allsyllabus.com
www.allsyllabus.com

166 j Network Theory

Making use of equation (3.2) in equation (3.1) leads to


3i2 3i2
i2 + 7 =
2
) i2 =
7
4
A
Thus, the total current
i = i1 + i2
7 5
=3 A= A
4 4
EXAMPLE 3.5
For the circuit shown in Fig. 3.14, find the terminal voltage Vab using superposition principle.

www.allsyllabus.com
SOLUTION Figure 3.14
As a first step in the analysis, deactivate the in-
dependent current source. This results in a cir-
cuit diagram as shown in Fig. 3.15.
Applying KVL clockwise gives
4 + 10 0+3 Vab1 + Vab1 = 0
) 4Vab1 = 4
) Vab1 = 1V
Figure 3.15
Next step in the analysis is to deactivate the
independent voltage source, resulting in a cir-
cuit diagram as shown in Fig. 3.16.
Applying KVL gives

10 2+3 Vab2 + Vab2 = 0


) 4Vab2 = 20
) Vab2 = 5V

Figure 3.16

vtu.allsyllabus.com
www.allsyllabus.com

Circuit Theorems j 167

According to superposition principle,

Vab = Vab1 + Vab2


= 1 + 5 = 6V

EXAMPLE 3.6
Use the principle of superposition to solve for vx in the circuit of Fig. 3.17.

www.allsyllabus.com Figure 3.17

SOLUTION
According to the principle of superposition,
vx = vx 1 + vx 2

where vx1 is produced by 6A source alone in the circuit and vx2 is produced solely by 4A current
source.
To find vx1 , deactivate the 4A current source. This results in a circuit diagram as shown in
Fig. 3.18.
KCL at node x1 :

vx 1 vx 1 4ix1
+ =6
2 8
vx 1
But ix1 =
2
v
vx 1 vx1 4 x21
Hence; + =6
2 8

) vx 1
2
+
vx 1
8
2vx1
=6

) 4vx1 + vx1 2vx1 = 48 Figure 3.18

) vx 1 =
48
3
= 16V

vtu.allsyllabus.com
www.allsyllabus.com

168 j Network Theory

To find vx2 , deactivate the 6A current source, resulting in a circuit diagram as shown in Fig.
3.19.
KCL at node x2 :

vx 2 ( 4ix2 )
v x2
+ =4
8 2
) vx 2
8
vx + 4ix2
+ 2
2
=4 (3.3)

Applying KVL along dotted path, we get

vx 2 + 4ix2 2ix2 = 0
) vx2 = 2ix2 or ix2 =
vx 2
2
(3.4)

Substituting equation (3.4) in equation (3.3), we get


 vx 2

vx 2 +4
vx 2 2

)
8www.allsyllabus.com
+
vx 2
+
2
vx 2 2vx2
=4

=4
8 2
) vx2 vx2
=4
)
8 2
vx 2 4vx2 = 32
) vx 2 =
32
3
V

Hence, according to the superposition principle,

vx = vx 1 + vx 2 Figure 3.19
32
= 16 = 5:33V
2
EXAMPLE 3.7
Which of the source in Fig. 3.20 contributes most of the power dissipated in the 2 Ω resistor ?
The least ? What is the power dissipated in 2 Ω resistor ?

Figure 3.20

vtu.allsyllabus.com
www.allsyllabus.com

Circuit Theorems j 169

SOLUTION
The Superposition theorem cannot be used to identify the individual contribution of each source
to the power dissipated in the resistor. However, the superposition theorem can be used to find the
total power dissipated in the 2 Ω resistor.

www.allsyllabus.com Figure 3.21

According to the superposition principle,


i1 = i01 + i02
where i01 = Contribution to i1 from 5V source alone.
and i02 = Contribution to i1 from 2A source alone.
Let us first find i01 . This needs the deactivation of 2A source. Refer to Fig. 3.22.
0
5
i1 = = 1:22A
2 + 2:1
Similarly to find i02 we have to disable the 5V source by shorting it.
Referring to Fig. 3.23, we find that
0
i2 =

2 2:1
= 1:024 A
2 + 2:1

Figure 3.22 Figure 3.23

vtu.allsyllabus.com
www.allsyllabus.com

170 j Network Theory

Total current,

i1 = i01 + i02
= 1:22 1:024
= 0:196 A
Thus; P2Ω = (0:196)2 2
= 0:0768 Watts
= 76:8 mW

EXAMPLE 3.8
Find the voltage V1 using the superposition principle. Refer the circuit shown in Fig.3.24.

www.allsyllabus.com
Figure 3.24

SOLUTION
According to the superposition principle,

V1 = V10 + V100

where V10 is the contribution from 60V source alone and V100 is the contribution from 4A current
source alone.
To find V10 , the 4A current source is opened, resulting in a circuit as shown in Fig. 3.25.

Figure 3.25

vtu.allsyllabus.com
www.allsyllabus.com

Circuit Theorems j 171

Applying KVL to the left mesh:

30ia 60 + 30 (ia ib ) =0 (3.5)


Also ib = 0:4iA
= 0:4 ( ia ) = 0:4ia (3.6)

Substituting equation (3.6) in equation (3.5), we get

30ia 60 + 30ia 
30 0:4ia = 0
) ia =
60
= 1:25A

48
ib = 0:4ia = 0:4 1:25
= 0:5A
Hence; V1
0
= (ia ib )  30
= 22:5 V

www.allsyllabus.com
To find, V100 , the 60V source is shorted as shown in Fig. 3.26.

Figure 3.26

Applying KCL at node a:


00
Va Va V1
+ =4
20 10
) 30Va 20V100 = 800 (3.7)

Applying KCL at node b:


00 00
V1 V1 Va
+ = 0:4ib
30 10
Also; Va = 20ia ) 20
ib =
Va

00 00
V1 V1 0:4Va
Va
Hence; + =
30 10 20
) 7:2Va + 8V100 = 0 (3.8)

vtu.allsyllabus.com
www.allsyllabus.com

172 j Network Theory

Solving the equations (3.7) and (3.8), we find that


00
V1 = 60V
Hence V1 = V10 + V100
= 22:5 + 60 = 82:5V

EXAMPLE 3.9
(a) Refer to the circuit shown in Fig. 3.27. Before the 10 mA current source is attached to
terminals x y , the current ia is found to be 1.5 mA. Use the superposition theorem to find
the value of ia after the current source is connected.
(b) Verify your solution by finding ia , when all the three sources are acting simultaneously.

www.allsyllabus.com

Figure 3.27

SOLUTION
According to the principle of superposition,

ia = ia1 + ia2 + ia3

where ia1 , ia2 and ia3 are the contributions to ia from 20V source, 5 mA source and 10 mA source
respectively.
As per the statement of the problem,

ia1 + ia2 = 1:5 mA

To find ia3 , deactivate 20V source and the 5 mA source. The resulting circuit diagram is


shown in Fig 3.28.
10mA 2k
ia3 = = 1 mA
18k + 2k
Hence, total current

ia = ia1 + ia2 + ia3


= 1:5 + 1 = 2:5 mA

vtu.allsyllabus.com
www.allsyllabus.com

Circuit Theorems j 173

Figure 3.28

(b) Refer to Fig. 3.29


KCL at node y:

18
Vy
 103
+
Vy
2 
20
103

= (10+5) 10 3

Solving, we get
www.allsyllabus.com
Vy
Vy = 45V:
45
Hence; ia =
18  10 3
=

18 103
= 2:5 mA
Figure 3.29
3.2 Thevenin’s theorem

In section 3.1, we saw that the analysis of a circuit may be greatly reduced by the use of su-
perposition principle. The main objective of Thevenin’s theorem is to reduce some portion of a
circuit to an equivalent source and a single element. This reduced equivalent circuit connected to
the remaining part of the circuit will allow us to find the desired current or voltage. Thevenin’s
theorem is based on circuit equivalence. A circuit equivalent to another circuit exhibits identical
characteristics at identical terminals.

Figure 3.30 A Linear two terminal network Figure 3.31 The Thevenin’s equivalent circuit

According to Thevenin’s theorem, the linear circuit of Fig. 3.30 can be replaced by the one
shown in Fig. 3.31 (The load resistor may be a single resistor or another circuit). The circuit to
the left of the terminals x y in Fig. 3.31 is known as the Thevenin’s equivalent circuit.

vtu.allsyllabus.com
www.allsyllabus.com

174 j Network Theory

The Thevenin’s theorem may be stated as follows:


A linear two–terminal circuit can be replaced by an equivalent circuit consisting of a
voltage source Vt in series with a resistor Rt , Where Vt is the open–circuit voltage at the termi-
nals and Rt is the input or equivalent resistance at the terminals when the independent sources
are turned off or Rt is the ratio of open–circuit voltage to the short–circuit current at the
terminal pair.
Action plan for using Thevenin’s theorem :

1. Divide the original circuit into circuit A and circuit B .

In general, circuit B is the load which may be linear or non-linear. Circuit A is the balance of
the original network exclusive of load and must be linear. In general, circuit A may contain
independent sources, dependent sources and resistors or other linear elements.

www.allsyllabus.com
2. Separate the circuit A from circuit B .
3. Replace circuit A with its Thevenin’s equivalent.
4. Reconnect circuit B and determine the variable of interest (e.g. current ‘i’ or voltage ‘v ’).

Procedure for finding Rt :


Three different types of circuits may be encountered in determining the resistance, Rt :
(i) If the circuit contains only independent sources and resistors, deactivate the sources and find
Rt by circuit reduction technique. Independent current sources, are deactivated by opening
them while independent voltage sources are deactivated by shorting them.

vtu.allsyllabus.com
www.allsyllabus.com

Circuit Theorems j 175

(ii) If the circuit contains resistors, dependent and independent sources, follow the instructions
described below:
(a) Determine the open circuit voltage voc with the sources activated.
(b) Find the short circuit current isc when a short circuit is applied to the terminals a b
voc
(c) Rt =
isc

(iii) If the circuit contains resistors and only dependent sources, then
(a) voc = 0 (since there is no energy source)
(b) Connect 1A current source to terminals
a b and determine vab .
vab
(c) Rt =
1
Figure 3.32

For all the cases discussed above, the Thevenin’s equivalent circuit is as shown in Fig. 3.32.

EXAMPLE 3.10
www.allsyllabus.com
Using the Thevenin’s theorem, find the current i through R = 2 Ω. Refer Fig. 3.33.

Figure 3.33

SOLUTION

Figure 3.34

vtu.allsyllabus.com
www.allsyllabus.com

176 j Network Theory

Since we are interested in the current i through R, the resistor R is identified as circuit B and
the remainder as circuit A. After removing the circuit B, circuit A is as shown in Fig. 3.35.

Figure 3.35

To find Rt , we have to deactivate the independent voltage source. Accordingly, we get the
circuit in Fig. 3.36.

Rt jj
= (5 Ω 20 Ω) + 4 Ω

=

5 20
+4=8Ω
www.allsyllabus.com
Referring to Fig. 3.35,
5 + 20 Rt

50 + 25I = 0 ) I = 2A
Figure 3.36
Hence Vab = Voc = 20(I ) = 40V

Thus, we get the Thevenin’s equivalent circuit which is as shown in Fig.3.37.

Figure 3.37 Figure 3.38

Reconnecting the circuit B to the Thevenin’s equivalent circuit as shown in Fig. 3.38, we get
40
i = = 4A
2+8

vtu.allsyllabus.com
www.allsyllabus.com

Circuit Theorems j 177

EXAMPLE 3.11
(a) Find the Thevenin’s equivalent circuit with respect to terminals a b for the circuit shown
in Fig. 3.39 by finding the open-circuit voltage and the short–circuit current.
(b) Solve the Thevenin resistance by removing the independent sources. Compare your result
with the Thevenin resistance found in part (a).

Figure 3.39
SOLUTION
www.allsyllabus.com

Figure 3.40
(a) To find Voc :
Apply KCL at node 2 :
V2 V2 30
+ 1:5 = 0
)
60 + 20 40
V2 = 60 Volts

Hence; Voc =I
 V 60 0 
=
2
60 + 20
60 
= 60
60
80
= 45 V

vtu.allsyllabus.com
www.allsyllabus.com

178 j Network Theory

To find isc :

Applying KCL at node 2:


V2 V2 30
+ 1:5 = 0
)
20 40
V2 = 30V
V2

Therefore;
www.allsyllabus.comisc

Rt
=

=
20
Voc
= 1:5A

=
45
isc 1:5
= 30 Ω
Figure 3.40 (a)

The Thevenin equivalent circuit with respect to the terminals a b is as shown in Fig. 3.40(a).
(b) Let us now find Thevenin resistance Rt by deactivating all the independent sources,

Rt Rt

Rt jj
= 60 Ω (40 + 20) Ω
60
= = 30 Ω (verified)
2

It is seen that, if only independent sources are present, it is easy to find Rt by deactivating all
the independent sources.

vtu.allsyllabus.com
www.allsyllabus.com

Circuit Theorems j 179

EXAMPLE 3.12
Find the Thevenin equivalent for the circuit shown in Fig. 3.41 with respect to terminals a b.

Figure 3.41

SOLUTION
To find Voc = Vab :
Applying KVL around the mesh of
Fig. 3.42, we get

)
20 + 6i www.allsyllabus.com
2i + 6i = 0
i = 2A

Since there is no current flowing in


10 Ω resistor, Voc = 6i = 12 V
To find Rt : (Refer Fig. 3.43)
Since both dependent and indepen- Figure 3.42
dent sources are present, Thevenin resis-
tance is found using the relation,
voc
Rt =
isc

Applying KVL clockwise for mesh 1 :

20 + 6i1 2i + 6 (i1 i2 ) =0
) 12i1 6i2 = 20 + 2i

Since i = i1 i2 , we get

12i1 6i2 = 20 + 2 (i1 i2 )

) 10i1 4i2 = 20

Applying KVL clockwise for mesh 2 :

10i2 + 6 (i2 i1 ) =0
) 6i1 + 16i2 = 0 Figure 3.43

vtu.allsyllabus.com
www.allsyllabus.com

180 j Network Theory

Solving the above two mesh equations, we get

i2 =
120
136
A ) isc = i2 =
120
136
A
voc 12
Rt = = = 13:6 Ω
isc 120
136
EXAMPLE 3.13
Find Vo in the circuit of Fig. 3.44 using Thevenin’s theorem.

www.allsyllabus.com Figure 3.44

SOLUTION
To find Voc :
Since we are interested in the voltage across 2 kΩ resistor, it is removed from the circuit of
Fig. 3.44 and so the circuit becomes as shown in Fig. 3.45.

Figure 3.45
By inspection, i1 = 4 mA
Applying KVL to mesh 2 :
12 + 6  10 3
(i2
 + 3  103 i2 = 0
i1 )
3

) 12 + 6  10 3
i2 4  10 3
+3  10 i2 =0

vtu.allsyllabus.com
www.allsyllabus.com

Circuit Theorems j 181

Solving, we get i2 = 4 mA
Applying KVL to the path 4 kΩ ! a b ! 3 kΩ, we get
4  10 3
i1 + Voc 3  10 = 0 3
i2

) Voc = 4  10 + 3  10
3
i1
3
i2

= 4  10  4  10 + 3  10  4  10
3 3 3 3
= 28V

To find Rt :
Deactivating all the independent sources, we get the circuit diagram shown in Fig. 3.46.

www.allsyllabus.com Figure 3.46

Rt = Rab = 4 kΩ + (6 kΩ 3 kΩ) = 6 kΩ jj
Hence, the Thevenin equivalent circuit is as shown in Fig. 3.47.

Figure 3.47 Figure 3.48

If we connect the 2 kΩ resistor to this equivalent network, we obtain the circuit of Fig. 3.48.

Vo =i 2  10  3

=
28
(6 + 2)  10
 2  10
3
3
= 7V

EXAMPLE 3.14
The wheatstone bridge in the circuit shown in Fig. 3.49 (a) is balanced when R2 = 1200 Ω. If the
galvanometer has a resistance of 30 Ω, how much current will be detected by it when the bridge
is unbalanced by setting R2 to 1204 Ω ?

vtu.allsyllabus.com
www.allsyllabus.com

182 j Network Theory

Figure 3.49(a)
SOLUTION
To find Voc :
We are interested in the galavanometer current. Hence, it is removed from the circuit of Fig.
3.49 (a) to find Voc and we get the circuit shown in Fig. 3.49 (b).
120 120
i1 = = A
900 + 600 1500
www.allsyllabus.com
i2 =
120
1204 + 800
=
120
2004
A

Applying KVL clockwise along the path


1204Ω ! b a !
900 Ω, we get
1204i2 Vt 900i1 = 0
) Vt = 1204i2900i1
= 1204 
120
2004
900  1500
120

= 95:8 mV
Figure 3.49(b)
To find Rt :
Deactivate all the independent sources and look into the terminals a b to determine the
Thevenin’s resistance.

Figure 3.49(c) Figure 3.49(d)

vtu.allsyllabus.com
www.allsyllabus.com

Circuit Theorems j 183

Rt jj
= Rab = 600 900 + 800 1204 jj
=

900 600 1204 800
+

1500 2004
= 840:64 Ω

Hence, the Thevenin equivalent circuit consists of the


95.8 mV source in series with 840.64Ω resistor. If we
connect 30Ω resistor (galvanometer resistance) to this
equivalent network, we obtain the circuit in Fig. 3.50. Figure 3.50

iG =

95:8 10 3
= 110:03 A
840:64 + 30 Ω

EXAMPLE 3.15
For the circuit shown in Fig. 3.51, find the Thevenin’s equivalent circuit between terminals a and b.

www.allsyllabus.com

Figure 3.51

SOLUTION
With ab shorted, let Isc = I . The circuit after
transforming voltage sources into their equiv-
alent current sources is as shown in Fig 3.52.
Writing node equations for this circuit,

At a : 0:2Va 0:1 Vc + I = 3
At c : 0:1Va + 0:3 Vc 0:1 Vb = 4
At b : 0:1Vc + 0:2 Vb I =1

As the terminals a and b are shorted Va = Vb


Figure 3.52
and the above equations become

vtu.allsyllabus.com
www.allsyllabus.com

184 j Network Theory

0:2Va 0:1 Vc + I = 3
0:2Va + 0:3 Vc = 4
0:2Va 0:1 Vc 1=1

Solving the above equations, we get the short circuit current, I = Isc = 1 A.
Next let us open circuit the terminals a and b and this makes I = 0. And the node equations
written earlier are modified to

0:2Va 0:1 Vc = 3
0:1Va + 0:3 Vc 0:1 Vb = 4
0:1Vc + 0:2 Vb = 1

Solving the above equations, we get

Va = 30V and Vb = 20V

www.allsyllabus.com
Hence, Vab = 30 20 = 10 V = Voc = Vt
Voc 10
Therefore Rt = = = 10Ω
Isc 1
The Thevenin’s equivalent is as shown in Fig 3.53
Figure 3.53

EXAMPLE 3.16
Refer to the circuit shown in Fig. 3.54. Find the Thevenin equivalent circuit at the terminals a b.

Figure 3.54

SOLUTION
To begin with let us transform 3 A current source and 10 V voltage source. This results in a
network as shown in Fig. 3.55 (a) and further reduced to Fig. 3.55 (b).

vtu.allsyllabus.com
www.allsyllabus.com

Circuit Theorems j 185

Figure 3.55(a)
Again transform the 30 V source and following the reduction procedure step by step from
Fig. 3.55 (b) to 3.55 (d), we get the Thevenin’s equivalent circuit as shown in Fig. 3.56.

www.allsyllabus.com

Figure 3.55(b) Figure 3.55(c)

Figure 3.55(d) Figure 3.56 Thevenin equivalent


circuit

EXAMPLE 3.17
Find the Thevenin equivalent circuit as seen from the terminals a b. Refer the circuit diagram
shown in Fig. 3.57.

vtu.allsyllabus.com
www.allsyllabus.com

186 j Network Theory

Figure 3.57
SOLUTION
Since the circuit has no independent sources, i = 0 when the terminals a b are open. There-
fore, Voc = 0.
The onus is now to find Rt . Since Voc = 0 and isc = 0, Rt cannot be determined from
Voc
Rt = . Hence, we choose to connect a source of 1 A at the terminals a b as shown in Fig.
isc
3.58. Then, after finding Vab , the Thevenin resistance is,
Vab
Rt =
1
KCL at node a : Va 2i Va
+ 1=0

Also;
www.allsyllabus.com 5
i =
10
Va

Va
 10
Va 2 10 Va
Hence; + 1=0
5 10
) Va =
50
13
V
50 Va
Hence; Rt =
Ω =
1 13
Alternatively one could find Rt by connecting a 1V source at the terminals a b and then find
1
the current from b to a. Then Rt = . The concept of finding Rt by connecting a 1A source
iba
between the terminals a b may also be used for circuits containing independent sources. Then
set all the independent sources to zero and use 1A source at the terminals a b to find Vab and
Vab
hence, Rt = .
1
For the present problem, the Thevenin equivalent circuit as seen between the terminals a b
is shown in Fig. 3.58 (a).

Figure 3.58 Figure 3.58 (a)

vtu.allsyllabus.com
www.allsyllabus.com

Circuit Theorems j 187

EXAMPLE 3.18
Determine the Thevenin equivalent circuit between the terminals a b for the circuit of Fig. 3.59.

Figure 3.59

SOLUTION
As there are no independent sources in the circuit, we get Voc = Vt = 0:
To find Rt , connect a 1V source to the terminals a b and measure the current I that flows
from b to a. (Refer Fig. 3.60 a).
1
Rt = Ω
www.allsyllabus.com I

Figure 3.60(a)

Applying KCL at node a:


Vx
I = 0:5Vx +
4
Since; Vx = 1V
1
we get, I = 0:5 + = 0:75 A
4 Figure 3.60(b)
1
Hence; Rt = = 1:33 Ω
0:75
The Thevenin equivalent circuit is shown in 3.60(b).
Alternatively, sticking to our strategy, let us connect 1A current source between the terminals
Vab
a b and then measure Vab (Fig. 3.60 (c)). Consequently, Rt = = Vab Ω:
1

vtu.allsyllabus.com
www.allsyllabus.com

188 j Network Theory

Applying KCL at node a:

0:5Vx +
Vx
4
=1 ) Vx = 1:33V
Vab Vx
Hence Rt = = = 1:33 Ω
1 1

The corresponding Thevenin equivalent


circuit is same as shown in Fig. 3.60(b) Figure 3.60(c)

3.3 Norton’s theorem

An American engineer, E.L. Norton at Bell Telephone Laboratories, proposed a theorem similar
to Thevenin’s theorem.
Norton’s theorem states that a linear two-terminal network can be replaced by an
equivalent circuit consisting of a current source iN in parallel with resistor RN , where iN

www.allsyllabus.com
is the short-circuit current through the terminals and RN is the input or equivalent resistance
at the terminals when the independent sources are turned off. If one does not wish to turn off
the independent sources, then RN is the ratio of open circuit voltage to short–circuit current
at the terminal pair.

Figure 3.61(a) Original circuit Figure 3.61(b) Norton’s equivalent circuit

Figure 3.61(b) shows Norton’s equivalent circuit as seen from the terminals a b of the
original circuit shown in Fig. 3.61(a). Since this is the dual of the Thevenin circuit, it is clear that
voc
RN = Rt and iN = . In fact, source transformation of Thevenin equivalent circuit leads to
Rt
Norton’s equivalent circuit.
Procedure for finding Norton’s equivalent circuit:
(1) If the network contains resistors and independent sources, follow the instructions below:
(a) Deactivate the sources and find RN by circuit reduction techniques.
(b) Find iN with sources activated.
(2) If the network contains resistors, independent and dependent sources, follow the steps given
below:
(a) Determine the short-circuit current iN with all sources activated.

vtu.allsyllabus.com
www.allsyllabus.com

Circuit Theorems j 189

(b) Find the open-circuit voltage voc .


voc
(c) Rt = RN =
iN

(3) If the network contains only resistors and dependent sources, follow the procedure
described below:

(a) Note that iN = 0.


(b) Connect 1A current source to the terminals a b and find vab .
vab
(c) Rt =
1
Note: Also, since vt = voc and iN = isc
voc
Rt = = RN
isc

The open–circuit and short–circuit test are sufficient to find any Thevenin or Norton equiva-
lent.

www.allsyllabus.com
3.3.1 PROOF OF THEVENIN’S AND NORTON’S THEOREMS

The principle of superposition is employed to provide the proof of Thevenin’s and Norton’s
theorems.

Derivation of Thevenin’s theorem:

Let us consider a linear circuit having two accessible terminals x y and excited by an external
current source i. The linear circuit is made up of resistors, dependent and independent sources. For
the sake of simplified analysis, let us assume that the linear circuit contains only two independent
voltage sources v1 and v2 and two independent current sources i1 and i2 . The terminal voltage v
may be obtained, by applying the principle of superposition. That is, v is made up of contributions
due to the external source and independent sources within the linear network.

Hence; v = a0 i + a1 v1 + a2 v2 + a3 i1 + a4 i2 (3.9)
= a0 i + b0 (3.10)
where b0 = a1 v1 + a2 v2 + a3 i1 + a4 i2
= contribution to the terminal voltage v by
independent sources within the linear network.

Let us now evaluate the values of constants a0 and b0 .

(i) When the terminals x and y are open–circuited, i = 0 and v = voc = vt . Making use of
this fact in equation 3.10, we find that b0 = vt .

vtu.allsyllabus.com
www.allsyllabus.com

190 j Network Theory

(ii) When all the internal sources are deactivated, b0 = 0. This enforces equation 3.10 to
become
v = a0 i = Rt i ) a0 = Rt
Rt

Vt

Figure 3.62 Current-driven circuit Figure 3.63 Thevenin’s equivalent circuit of Fig. 3.62

where Rt is the equivalent resistance of the linear network as viewed from the terminals x y .
Also, a0 must be Rt in order to obey the ohm’s law. Substuting the values of a0 and b0 in equation
3.10, we find that
v = Rt i + v1

www.allsyllabus.com
which expresses the voltage-current relationship at terminals
Thus, the two circuits of Fig. 3.62 and 3.63 are equivalent.
x y of the circuit in Fig. 3.63.

Derivation of Norton’s theorem:

Let us now assume that the linear circuit described earlier is driven by a voltage source v as shown
in Fig. 3.64.
The current flowing into the circuit can be obtained by superposition as

i = c0 v + d 0 (3.11)

where c0 v is the contribution to i due to the external voltage source v and d0 contains the contri-
butions to i due to all independent sources within the linear circuit. The constants c0 and d0 are
determined as follows :
(i) When terminals x y are short-circuited, v =
0 and i = isc . Hence from equation (3.11),
we find that i = d0 = isc , where isc is the
short-circuit current flowing out of terminal x,
which is same as Norton current iN

Thus, d0 = iN
Figure 3.64
Voltage-driven circuit

(ii) Let all the independent sources within the linear network be turned off, that is d0 = 0. Then,
equation (3.11) becomes
i = c0 v

vtu.allsyllabus.com
www.allsyllabus.com

Circuit Theorems j 191

For dimensional validity, c0 must have the


dimension of conductance. This enforces c0 =
1
where Rt is the equivalent resistance of the
Rt
linear network as seen from the terminals x y.
Thus, equation (3.11) becomes
1
i = v isc
Rt
Figure 3.65 Norton’s equivalent of
1
= v iN voltage driven circuit
Rt

This expresses the voltage-current relationship at the terminals x y of the circuit in Fig.
(3.65), validating that the two circuits of Figs. 3.64 and 3.65 are equivalents.

EXAMPLE 3.19
Find the Norton equivalent for the circuit of Fig. 3.66.

www.allsyllabus.com

Figure 3.66

SOLUTION
As a first step, short the terminals a b. This
results in a circuit diagram as shown in Fig. 3.67.
Applying KCL at node a, we get
0 24
3 + isc = 0
)
4
isc = 9A

To find RN , deactivate all the independent


sources, resulting in a circuit diagram as shown
in Fig. 3.68 (a). We find RN in the same way as Figure 3.67
Rt in the Thevenin equivalent circuit.

RN =
4 12
=3Ω

4 + 12

vtu.allsyllabus.com
www.allsyllabus.com

192 j Network Theory

Figure 3.68(a) Figure 3.68(b)

Thus, we obtain Nortion equivalent circuit as shown in Fig. 3.68(b).

EXAMPLE 3.20
Refer the circuit shown in Fig. 3.69. Find the value of ib using Norton equivalent circuit. Take
R = 667 Ω.

www.allsyllabus.com
Figure 3.69
SOLUTION
Since we want the current flowing through R, remove
R from the circuit of Fig. 3.69. The resulting circuit
diagram is shown in Fig. 3.70.
To find iac or iN referring Fig 3.70(a) :
0
ia = = 0A
1000
12
isc = A = 2 mA Figure 3.70
6000

Figure 3.70(a)

vtu.allsyllabus.com
www.allsyllabus.com

Circuit Theorems j 193

To find RN :
The procedure for finding RN is same that of Rt
in the Thevenin equivalent circuit.
voc
Rt = RN =
isc

To find voc , make use of the circuit diagram shown


in Fig. 3.71. Do not deactivate any source.
Applying KVL clockwise, we get
Figure 3.71
12 + 6000ia + 2000ia + 1000ia = 0
) ia =
4
3000
A

) voc = ia  4
1000 = V
3
4
voc

Therefore; RN = = 3 = 667 Ω
isc 2 10 3

www.allsyllabus.com
The Norton equivalent circuit along with resistor R is as shown below:

ib =
isc
=
2mA
= 1mA
2 2

Figure : Norton equivalent circuit with load R

EXAMPLE 3.21
Find Io in the network of Fig. 3.72 using Norton’s theorem.

Figure 3.72

vtu.allsyllabus.com
www.allsyllabus.com

194 j Network Theory

SOLUTION
We are interested in Io , hence the 2 kΩ resistor is removed from the circuit diagram of Fig. 3.72.
The resulting circuit diagram is shown in Fig. 3.73(a).

Figure 3.73(a) Figure 3.73(b)

To find iN or isc :

www.allsyllabus.com
Refer Fig. 3.73(b). By inspection, V1 = 12 V
Applying KCL at node V2 :

V2 V1 V2 V2 V1
+ + =0
6 kΩ 2 kΩ 3 kΩ
Substituting V1 = 12 V and solving, we get

V2 = 6V
V1 V2 V1
isc = + = 5 mA
3 kΩ 4 kΩ
To find RN :
Deactivate all the independent sources (refer Fig. 3.73(c)).

Figure 3.73(c) Figure 3.73(d)

vtu.allsyllabus.com
www.allsyllabus.com

Circuit Theorems j 195

Referring to Fig. 3.73 (d), we get

RN jj jj
= Rab = 4 kΩ [3 kΩ + (6 kΩ 2 kΩ)] = 2:12 kΩ

Hence, the Norton equivalent circuit


along with 2 kΩ resistor is as shown in
Fig. 3.73(e).

Io =
isc  RN
= 2:57mA
R + RN
Figure 3.73(e)

EXAMPLE 3.22
Find Vo in the circuit of Fig. 3. 74.

www.allsyllabus.com

Figure 3.74

SOLUTION
Since we are interested in Vo , the voltage across 4 kΩ resistor, remove this resistance from the
circuit. This results in a circuit diagram as shown in Fig. 3.75.

Figure 3.75

vtu.allsyllabus.com
www.allsyllabus.com

196 j Network Theory

To find isc , short the terminals a b :

www.allsyllabus.com

vtu.allsyllabus.com
www.allsyllabus.com

Circuit Theorems j 197

Constraint equation :
i1 i2 = 4mA (3.12)
KVL around supermesh :
4+2  10 3
i1 +4  10 3
i2 =0 (3.13)
KVL around mesh 3 :
8  10 ( 3
i3 i2 ) +2  10 ( 3
i3 i1 ) =0
Since i3 = isc , the above equation becomes,
8  10 ( 3
isc i2 ) +2  10 ( 3
isc i1 ) =0 (3.14)
Solving equations (3.12), (3.13) and (3.14) simultaneously, we get isc = 0:1333 mA.
To find RN :
Deactivate all the sources in Fig. 3.75. This yields a circuit diagram as shown in Fig. 3.76.

www.allsyllabus.com

Figure 3.76

RN = 6 kΩ 10 kΩjj
=

6 10
= 3:75 kΩ
6 + 10
Hence, the Norton equivalent circuit is as shown
in Fig 3.76 (a).
To the Norton equivalent circuit, now connect the
4 kΩ resistor that was removed earlier to get the Figure 3.76(a)
network shown in Fig. 3.76(b).

vtu.allsyllabus.com
www.allsyllabus.com

198 j Network Theory

Vo = isc (RN jj R)

RN R
= isc
RN +R
= 258 mV

Figure 3.76(b) Norton equivalent circuit with R = 4 kΩ

EXAMPLE 3.23
Find the Norton equivalent to the left of the terminals a b for the circuit of Fig. 3.77.

www.allsyllabus.com
Figure 3.77

SOLUTION
To find isc :

Note that vab = 0 when the terminals a b are short-circuited.


5
Then i == 10 mA
500
Therefore, for the right–hand portion of the circuit, isc = 10i = 100 mA.

vtu.allsyllabus.com
www.allsyllabus.com

Circuit Theorems j 199

To find RN or Rt :

Writing the KVL equations for the left-hand mesh, we get


5 + 500i + vab = 0 (3.15)
Also for the right-hand mesh, we get
vab = 25(10i) = 250i
vab
Therefore
www.allsyllabus.com i
250
=

Substituting i into the mesh equation (3.15), we get


 vab

5 + 500 + vab = 0
250
) vab= 5V
voc vab 5
RN = Rt  = = = 50 Ω
isc isc 0:1
The Norton equivalent circuit is shown in
Fig 3.77 (a).

Figure 3.77 (a)

EXAMPLE 3.24
Find the Norton equivalent of the network shown in Fig. 3.78.

Figure 3.78

vtu.allsyllabus.com
www.allsyllabus.com

200 j Network Theory

SOLUTION
Since there are no independent sources present in the network of Fig. 3.78, iN = isc = 0.
To find RN , we inject a current of 1A between the terminals a b. This is illustrated in
Fig. 3.79.

Figure 3.79 Figure 3.79(a) Norton


equivalent circuit

KCL at node 1:

)
www.allsyllabus.com 1=
v1
100
0:03v1
+
v1
50
v2

0:02v2 = 1
KCL at node 2: v2 v2 v1
+ + 0:1v1 = 0
)
200 50
0:08v1 + 0:025v2 = 0
Solving the above two nodal equations, we get
v1 = 10:64 volts )
voc = 10:64 volts
10:64
voc
Hence; R N = Rt = = = 10:64 Ω
1 1
Norton equivalent circuit for the network shown in Fig. 3.78 is as shown in Fig. 3.79(a).

EXAMPLE 3.25
Find the Thevenin and Norton equivalent circuits for the network shown in Fig. 3.80 (a).

Figure 3.80(a)

vtu.allsyllabus.com
www.allsyllabus.com

Circuit Theorems j 201

SOLUTION
To find Voc :
Performing source transformation on 5A current source, we get the circuit shown in
Fig. 3.80 (b).
Applying KVL around Left mesh :
50 + 2ia 20 + 4ia = 0
) ia =
70
6
A

Applying KVL around right mesh:

20 + 10ia + Voc 4ia = 0


) Voc = 90 V
Figure 3.80(b)

To find isc (referring Fig 3.80 (c)) :


KVL around Left mesh :
50 + 2ia
)
www.allsyllabus.com
20 + 4 (ia
6ia 4isc = 70
isc ) =0

KVL around right mesh :

4 (isc ia ) + 20 + 10ia = 0
) 6ia + 4isc = 20

Figure 3.80(c)

Solving the two mesh equations simultaneously, we get isc = 11:25 A


voc 90
Hence, Rt = RN = = =8Ω
isc 11:25
Performing source transformation on Thevenin equivalent circuit, we get the norton equivalent
circuit (both are shown below).

Thevenin equivalent circuit Norton equivalent circuit

vtu.allsyllabus.com
www.allsyllabus.com

202 j Network Theory

EXAMPLE 3.26
If an 8 kΩ load is connected to the terminals of the
network in Fig. 3.81, VAB = 16 V. If a 2 kΩ load is
connected to the terminals, VAB = 8V. Find VAB if a
20 kΩ load is connected across the terminals.

SOLUTION
Figure 3.81

www.allsyllabus.com
Applying KVL around the mesh, we get (Rt + RL ) I = Voc

If RL = 2 kΩ; I = 10 mA ) Voc = 20 + 0:01Rt


If RL = 10 kΩ; I = 6 mA ) Voc = 60 + 0:006Rt

Solving, we get Voc = 120 V, Rt = 10 kΩ.


Voc 120
If RL = 20 kΩ; I =
(RL + Rt )
=
(20  103 + 10  10 ) = 4 mA
3

3.4 Maximum Power Transfer Theorem

In circuit analysis, we are some times interested


in determining the maximum power that a circuit
can supply to the load. Consider the linear circuit
A as shown in Fig. 3.82.
Circuit A is replaced by its Thevenin equivalent
circuit as seen from a and b (Fig 3.83).
We wish to find the value of the load RL such that Figure 3.82 Circuit A with load RL
the maximum power is delivered to it.
The power that is delivered to the load is given by

 Vt
2
2
p =i RL = RL (3.16)
Rt + RL

vtu.allsyllabus.com
www.allsyllabus.com

Circuit Theorems j 203

Assuming that Vt and Rt are fixed for a given source, the maximum power is a function of
RL .In order to determine the value of RL that maximizes p, we differentiate p with respect to
RL and equate the derivative to zero.
" #
dp (Rt + RL )2 2 (Rt + RL )
= Vt2 =0
dRL (RL + Rt )2
which yields RL = Rt (3.17)

To confirm that equation (3.17) is a maximum,


2
d p
it should be shown that 2 < 0. Hence, maxi-
dRL
mum power is transferred to the load when RL is
equal to the Thevenin equivalent resistance Rt .
The maximum power transferred to the load is
obtained by substituting RL = Rt in equation
3.16.
Accordingly, Figure 3.83 Thevenin equivalent circuit

www.allsyllabus.com
Pmax =
2
Vt R L
=
Vt
2
is substituted for circuit A

(2RL )2 4RL

The maximum power transfer theorem states that the maximum power delivered by a source
represented by its Thevenin equivalent circuit is attained when the load RL is equal to the
Thevenin resistance Rt .

EXAMPLE 3.27
Find the load RL that will result in maximum power delivered to the load for the circuit of Fig.
3.84. Also determine the maximum power Pmax .

Figure 3.84

SOLUTION
Disconnect the load resistor RL . This results in a circuit diagram as shown in Fig. 3.85(a).
Next let us determine the Thevenin equivalent circuit as seen from a b.

vtu.allsyllabus.com
www.allsyllabus.com

204 j Network Theory

180
i = = 1A

150 + 30
Voc = Vt = 150 i = 150 V

To find Rt , deactivate the 180 V source. This results in the


circuit diagram of Fig. 3.85(b).

Rt = Rab = 30 Ω 150 Ωjj


=
30 150  = 25 Ω
Figure 3.85(a)

30 + 150
The Thevenin equivalent circuit connected to the
load resistor is shown in Fig. 3.86.
Maximum power transfer is obtained when
RL = Rt = 25 Ω:
Then the maximum power is

(150)2
2

www.allsyllabus.com

Pmax
4RL
=
Vt
4 25
=
= 2:25 Watts
Figure 3.85(b)

The Thevenin source Vt actually provides a total


power of

Pt = 150  i

= 150 
150
25 + 25
= 450 Watts
Thus, we note that one-half the power is dissipated in RL .
Figure 3.86
EXAMPLE 3.28
Refer to the circuit shown in Fig. 3.87. Find the value of RL for maximum power transfer. Also
find the maximum power transferred to RL .

Figure 3.87

vtu.allsyllabus.com
www.allsyllabus.com

Circuit Theorems j 205

SOLUTION
Disconnecting RL , results in a circuit diagram as shown in Fig. 3.88(a).

Figure 3.88(a)

To find Rt , deactivate all the independent voltage sources as in Fig. 3.88(b).

www.allsyllabus.com

Figure 3.88(b) Figure 3.88(c)

Rt jj
= Rab = 6 kΩ 6 kΩ 6 kΩ jj
= 2 kΩ

To find Vt :
Refer the Fig. 3.88(d).
Constraint equation :

V3 V1 = 12 V

By inspection, V2 =3V
KCL at supernode :
V3 V2 V1 V1 V2
+ + =0
6k 6k 6k

) V3
6k
3
+
V3
6k
12
+
V3 12 3
6k
=0
Figure 3.88(d)

vtu.allsyllabus.com
www.allsyllabus.com

206 j Network Theory

) V3 3 + V3 12 + V3 15 = 0
) 3V3 = 30
) V3 = 10
) Vt = Vab = V3 = 10 V

Figure 3.88(e)

The Thevenin equivalent circuit connected to the load resistor RL is shown in Fig. 3.88(e).

Pmax = i2 RL
 Vt
2
= RL
2RL
= 12:5 mW

Alternate method :
www.allsyllabus.com
It is possible to find Pmax , without finding the Thevenin equivalent circuit. However, we have to
find Rt . For maximum power transfer, RL = Rt = 2 kΩ. Insert the value of RL in the original
circuit given in Fig. 3.87. Then use any circuit reduction technique of your choice to find power
dissipated in RL .
Refer Fig. 3.88(f). By inspection we find that, V2 = 3 V.
Constraint equation :

V3 V1 = 12
) V1 = V3 12

KCL at supernode :
V3 V2 V1 V2 V3 V1
+ + + =0
6k 6k 2k 6k

) V3
6k
3
+
V3 12
6k
3
+
2k
+
V3
6k
=0
V3 12

) V3 3 + V3 15 + 3V3 + V3 12 = 0
) 6V3 = 30
) V3 =5 V Figure 3.88(f)

V3
2 25
Hence; Pmax = = = 12:5 mW
RL 2k

vtu.allsyllabus.com
www.allsyllabus.com

Circuit Theorems j 207

EXAMPLE 3.29
Find RL for maximum power transfer and the maximum power that can be transferred in the
network shown in Fig. 3.89.

Figure 3.89

SOLUTION
www.allsyllabus.com
Disconnect the load resistor RL . This results in a circuit as shown in Fig. 3.89(a).

Figure 3.89(a)

To find Rt , let us deactivate all the independent sources, which results the circuit as shown in
Fig. 3.89(b).
Rt = Rab = 2 kΩ + 3 kΩ + 5 kΩ = 10 kΩ

For maximum power transfer RL = Rt = 10 kΩ.


Let us next find Voc or Vt .
Refer Fig. 3.89 (c). By inspection, i1 = 2 mA & i2 = 1 mA.

vtu.allsyllabus.com
www.allsyllabus.com

208 j Network Theory

Figure 3.89(b)

Applying KVL clockwise to the loop 5 kΩ ! 3 kΩ ! 2 kΩ ! , we get a b

5k  + 3k (
i2 ) + 2k  + = 0
i1 i2 i1 Vt
  
) 5  10 1  10
3 3 +3  1032  10 1  10 +2  10
3 2  10 3 3 3 + Vt = 0
) 5 9 4 + Vt = 0
) Vt = 18 V:
The Thevenin equivalent circuit with load resistor RL is as shown in Fig. 3.89 (d).
18
i =
(10 + 10) 103
= 0:9 mA

Then,
Pmax = PL = (0:9 mA)2  10 kΩ
= 8:1 mW
www.allsyllabus.com

Figure 3.89(c) Figure 3.89(d)

EXAMPLE 3.30
Find the maximum power dissipated in RL . Refer the circuit shown in Fig. 3.90.

Figure 3.90

vtu.allsyllabus.com
www.allsyllabus.com

Circuit Theorems j 209

SOLUTION
Disconnecting the load resistor RL from the original circuit results in a circuit diagram as shown
in Fig. 3.91.

Figure 3.91
As a first step in the analysis, let us find Rt . While finding Rt , we have to deactivate all the
independent sources. This results in a network as shown in Fig 3.91 (a) :

www.allsyllabus.com

Figure 3.91(a)

Rt jj
= Rab = [140 Ω 60 Ω] + 8 Ω
=

140 60
+ 8 = 50 Ω:
140 + 60
For maximum power transfer, RL = Rt = 50 Ω. Next step in the analysis is to find Vt .
Refer Fig 3.91(b), using the principle of
current division,

i1 =

i R2
+
R1 R2

20  170
= = 17 A
170 + 30

i2 =
 = 20  30
i R1
R1 + R2 170 + 30
600
= = 3A
200
Figure 3.91(a)

vtu.allsyllabus.com
www.allsyllabus.com

210 j Network Theory

Applying KVL clockwise to the loop comprising of 50 Ω ! 10 Ω ! 8 Ω ! a b, we get

50i2 10i1 + 8 0+ Vt =0


) 50(3) 10 (17) + Vt = 0
) Vt = 20 V

The Thevenin equivalent circuit with load resistor RL is


as shown in Fig. 3.91(c).
Figure 3.91(c)
20
iT = = 0:2A
50 + 50
2
Pmax = iT  
50 = 0:04 50 = 2 W

EXAMPLE 3.31
Find the value of RL for maximum power transfer in the circuit shown in Fig. 3.92. Also
find Pmax .

www.allsyllabus.com

Figure 3.92

SOLUTION
Disconnecting RL from the original circuit, we get the network shown in Fig. 3.93.

Figure 3.93

vtu.allsyllabus.com
www.allsyllabus.com

Circuit Theorems j 211

Let us draw the Thevenin equivalent circuit as seen from the terminals a b and then insert
the value of RL = Rt between the terminals a b. To find Rt , let us deactivate all independent
sources which results in the circuit as shown in Fig. 3.94.

Figure 3.94

Rt = Rab
jj =8Ω 2Ω
www.allsyllabus.com
 =
8 2
8+2
= 1:6 Ω

Next step is to find Voc or Vt .


By performing source transformation on the circuit shown in Fig. 3.93, we obtain the circuit
shown in Fig. 3.95.

Figure 3.95

Applying KVL to the loop made up of 20 V ! 3 Ω ! 2 Ω ! 10 V ! 5 Ω ! 30 V, we get


20 + 10i 10 30 = 0
) i =
60
10
= 6A

vtu.allsyllabus.com
www.allsyllabus.com

212 j Network Theory

Again applying KVL clockwise to the path 2 Ω ! 10 V ! a b, we get


2i 10 Vt = 0
) Vt = 2i 10
= 2(6) 10 = 2 V

The Thevenin equivalent circuit with load resistor


RL is as shown in Fig. 3.95 (a).

Pmax = i2T RL
Vt
2
Figure 3.95(a) Thevenin equivalent
= = 625 mW
4Rt circuit

EXAMPLE 3.32
Find the value of RL for maximum power transfer. Hence find Pmax .

www.allsyllabus.com
Figure 3.96
SOLUTION
Removing RL from the original circuit gives us the circuit diagram shown in Fig. 3.97.

Figure 3.97

To find Voc :
KCL at node A :
0
ia 0:9 + 10i0a = 0
) 0
ia = 0:1 A

Hence; Voc = 3 10i0a
=3  10  0 1 = 3 V
:

vtu.allsyllabus.com
www.allsyllabus.com

Circuit Theorems j 213

To find Rt , we need to compute isc with all independent sources activated.


KCL at node A:
00
ia 0:9 + 10ia 00 = 0
) ia
00
= 0:1 A
Hence isc = 10ia = 10 00
01=1A
:
Voc 3
Rt = = =3Ω
isc 1
Hence, for maximum power transfer RL = Rt = 3 Ω.
The Thevenin equivalent circuit with RL = 3 Ω
inserted between the terminals a b gives the net-
work shown in Fig. 3.97(a).

3
iT = = 0:5 A
3+3
Pmax = i2T RL

www.allsyllabus.com = (0:5)2 3
= 0.75 W
Figure 3.97(a)
EXAMPLE 3.33
Find the value of RL in the network shown that will achieve maximum power transfer, and deter-
mine the value of the maximum power.

Figure 3.98(a)
SOLUTION
Removing RL from the circuit of Fig. 3.98(a), we
get the circuit of Fig 3.98(b).
Applying KVL clockwise we get

12 + 2 103 i + 2Vx0 = 0
Also 0
Vx =1  10 3
i


Hence; 12 + 2  10 3i +2 1  10 3i =0
12
4  10
i = = 3 mA Figure 3.98(b)
3

vtu.allsyllabus.com
www.allsyllabus.com

214 j Network Theory

Applying KVL to loop 1 kΩ !2 !


Vx

b a, we get

1  10 3
i + 2Vx0 Vt =0

) Vt =1  10 + 2 1  10
3
i
3
i

= 1  10 + 2  10 3 3
 i

= 3  10 3  10 3 3

=9V

To find Rt , we need to find isc . While finding isc ,


none of the independent sources must be deacti-
vated.
Applying KVL to mesh 1:

12 + Vx 00 + 0 = 0
) Vx
00
= 12
)  ) 3

www.allsyllabus.com
1

Applying KVL to mesh 2:


10 i1 = 12 i1 = 12 mA

1  10
3
i2 + 2Vx 00 = 0
) 1  10 3
i2 = 24
i2 = 24 mA
Applying KCL at node a:

isc = i1 i2

= 12 + 24 = 36 mA
Vt Voc
Hence; Rt = =
isc isc
9
=
36 10  3
= 250 Ω

For maximum power transfer, RL = Rt = 250 Ω.


Thus, the Thevenin equivalent circuit with RL is
as shown in Fig 3.98 (c) :
9 9
iT = = A
250 + 250 500
2
Pmax = iT 250


=
9 2
500
 250 Figure 3.98 (c) Thevenin equivalent circuit
= 81 mW
vtu.allsyllabus.com
www.allsyllabus.com

Circuit Theorems j 215

EXAMPLE 3.34
The variable resistor RL in the circuit of Fig. 3.99 is adjusted untill it absorbs maximum power
from the circuit.
(a) Find the value of RL .
(b) Find the maximum power.

Figure 3.99

SOLUTION

Fig. 3.99(a).
www.allsyllabus.com
Disconnecting the load resistor RL from the original circuit, we get the circuit shown in

Figure 3.99(a)
KCL at node v1 :

v1 100 v1 13i0a v1 v2
+ + =0 (3.18)
2 5 4
Constraint equations :
0
100 v1
ia = (3.19)
2
v2 v1
= va0 (applying K C L at v2 ) (3.20)
4
0
va = v1 v2 (potential across 4 Ω) (3.21)

vtu.allsyllabus.com
www.allsyllabus.com

216 j Network Theory

From equations (3.20) and (3.21), we have


v2 v1
= v1 v2

)
4
v2 v1 = 4v1 4v2
) 5v1 5v2 = 0
) v1 = v2 (3.22)

Making use of equations (3.19) and (3.22) in (3.18), we get


(100 v1 )
v1 100 v2 13 v1 v1
+ 2 + =0
2 5 4
 
) 5( 100) + 2 13
v1
2
=0 v1
(100 v1 )

) 5 500 + 2 13  100 + 13 = 0
v1 v1 v1

) 20 = 1800 v1

)
www.allsyllabus.com
= 90 Volts v1

Hence; vt = v2 = v1 = 90 Volts
voc vt
We know that, Rt = =
isc isc

The short circuit current is calculated using the circuit shown below:

00
100 v1
Here ia =
2
Applying KCL at node v1 :

v1 100 v1 13ia v1 0
+ + =0
2 5 4
(100 v1 )
) v1 100 v1 13 v1
+ 2 + =0
2 5 4

vtu.allsyllabus.com
www.allsyllabus.com

Circuit Theorems j 217

Solving we get v1 = 80 volts = va00


Applying KCL at node a :
0 v1
+ isc = va00
4
) isc
4
=
v1
+ va00
80
= + 80 = 100 A
4
voc vt
Hence; Rt = =
isc isc
90
= = 0:9 Ω
100
Hence for maximum power transfer,

RL = Rt = 0:9 Ω

The Thevenin equivalent circuit with RL = 0:9 Ω


is as shown.
90
www.allsyllabus.com
90
it = =
0:9 + 0:9 1:8
2
Pmax = it 0:9 
 90 2
=
1:8
 0 9 = 2250 W
:

EXAMPLE 3.35
Refer to the circuit shown in Fig. 3.100 :
(a) Find the value of RL for maximum power transfer.
(b) Find the maximum power that can be delivered to RL .

Figure 3.100

vtu.allsyllabus.com
www.allsyllabus.com

218 j Network Theory

SOLUTION
Removing the load resistor RL , we get the circuit diagram shown in Fig. 3.100(a). Let us proceed
to find Vt .

Figure 3.100(a)

www.allsyllabus.com
Constraint equation :
ia
0
= i1 i3

KVL clockwise to mesh 1 :


200 + 1 (i1 i2 ) + 20 (i1 i3 ) + 4i1 = 0
) 25i1 i2 20i3 = 200
KVL clockwise to mesh 2 :
14i0a + 2 (i2 i3 ) + 1 (i2 i1 ) =0
) 14 (i1 i3 ) + 2 (i2 i3 ) + 1 (i2 i1 ) =0
) 13i1 + 3i2 16i3 = 0
KVL clockwise to mesh 3 :
2 (i3 i2 ) 100 + 3i3 + 20 (i3 i1 ) =0
) 20i1 2i2 + 25i3 = 100
Solving the mesh equations, we get
i1 = 2:5A; i3 = 5A
Applying KVL clockwise to the path comprising of a b ! 20 Ω, we get
Vt 20i0a = 0
) Vt = 20i0a
= 20 (i1 i3 )

= 20 ( 2:5 5)
= 150 V

vtu.allsyllabus.com
www.allsyllabus.com

Circuit Theorems j 219

Next step is to find Rt .


Voc Vt
Rt = =
isc isc

When terminals a b are shorted, i00a = 0. Hence, 14 i00a is also zero.

www.allsyllabus.com

KVL clockwise to mesh 1 :

200 + 1 (i1 i2 ) + 4i1 = 0


) 5i1 i2 = 200

KVL clockwise to mesh 2 :

2 (i2 i3 ) + 1 (i2 i1 ) =0
) i1 + 3i2 2i3 = 0

KVL clockwise to mesh 3 :

100 + 3i3 + 2 (i3 i2 ) =0


) 2i2 + 5i3 = 100

vtu.allsyllabus.com
www.allsyllabus.com

220 j Network Theory

Solving the mesh equations, we find that


i1 = 40A; i3 = 20A;
) i3 = 60A
isc = i1
150 Vt
Rt = = = 2:5 Ω
isc 60
For maximum power transfer, RL = Rt = 2:5 Ω. The Thevenin equivalent circuit with RL is
as shown below :

www.allsyllabus.com
 

Pmax = i21 RL

=
150 2
2:5
2:5 + 2:5
= 2250 W
EXAMPLE 3.36
A practical current source provides 10 W to a 250 Ω load and 20 W to an 80 Ω load. A resistance
RL , with voltage vL and current iL , is connected to it. Find the values of RL , vL and iL if
(a) vL iL is a maximum, (b) vL is a maximum and (c) iL is a maximum.
SOLUTION
Load current calculation:
r
10
10W to 250 Ω corresponds to iL =
250
=r 200 mA
20
20W to 80 Ω corresponds to iL =
80
= 500 mA
Using the formula for division of current between two parallel branches :

i2 =
i  R1
R1 + R2
IN R N
In the present context, 0:2 = (3.23)
RN + 250
IN R N
and 0:5 = (3.24)
RN + 80

vtu.allsyllabus.com
www.allsyllabus.com

Circuit Theorems j 221

Solving equations (3.23) and (3.24), we get

IN = 1:7 A
RN = 33:33 Ω

(a) If vL iL is maximum,

RL = RN = 33:33 Ω
iL = 1:7  33:33
33:33 + 33:33
= 850 mA
vL = iL RL = 850  10  33 33
3
:

= 28:33 V
(b) vL = IN (RN jj RL ) is a maximum when RN RL jj is a maximum, which occurs when
RL = . 1
Then, iL = 0 and

www.allsyllabus.com vL = 1:7 RN

 = 1:7 33:33
= 56:66 V
IN R N
(c) iL = is maxmimum when RL = 0 Ω
RN + RL
) iL = 1:7A and vL = 0 V

3.5 Sinusoidal steady state analysis using superposition, Thevenin and


Norton equivalents

Circuits in the frequency domain with phasor currents and voltages and impedances are analogous
to resistive circuits.
To begin with, let us consider the principle of superposition, which may be restated as follows :
For a linear circuit containing two or more independent sources, any circuit voltage or
current may be calculated as the algebraic sum of all the individual currents or voltages caused
by each independent source acting alone.

Figure 3.101 Thevenin equivalent circuit Figure 3.102 Norton equivalent circuit

vtu.allsyllabus.com
www.allsyllabus.com

222 j Network Theory

The superposition principle is particularly useful if a circuit has two or more sources acting
at different frequencies. The circuit will have one set of impedance values at one frequency and a
different set of impedance values at another frequency. Phasor responses corresponding to differ-
ent frequencies cannot be superposed; only their corresponding sinusoids can be superposed. That
is, when frequencies differ, the principle of superposition applies to the summing of time domain
components, not phasors. Within a component, problem corresponding to a single frequency,
however phasors may be superposed.
Thevenin and Norton equivalents in phasor circuits are found exactly in the same manner
as described earlier for resistive circuits, except for the subtitution of impedance Z in place of
resistance R and subsequent use of complex arithmetic. The Thevenin and Norton equivalent
circuits are shown in Fig. 3.101 and 3.102.
The Thevenin and Norton forms are equivalent if the relations

(a) Zt = ZN (b) Vt = ZN IN

hold between the circuits.


A step by step procedure for finding the Thevenin equivalent circuit is as follows:

www.allsyllabus.com
1. Identify a seperate circuit portion of a total circuit.
2. Find Vt = Voc at the terminals.
3. (a) If the circuit contains only impedances and independent sources, then deactivate all the
independent sources and then find Zt by using circuit reduction techniques.
(b) If the circuit contains impedances, independent sources and dependent sources, then
either short–circuit the terminals and determine Isc from which
Voc
Zt =
Isc
or deactivate the independent sources, connect a voltage or current source at the terminals, and
determine both V and I at the terminals from which
V
Zt =
I
A step by step procedure for finding Norton equivalent circuit is as follows:

(i) Identify a seperate circuit portion of the original circuit.


(ii) Short the terminals after seperating a portion of the original circuit and find the current
through the short circuit at the terminals, so that IN = Isc .
(iii) (a) If the circuit contains only impedances and independent sources, then deactivate all the
independent sources and then find ZN = Zt by using circuit reduction techniques.
(b) If the circuit contains impedances, independent sources and one or more dependent
Voc
sources, find the open–circuit voltage at the terminals, Voc , so that ZN = Zt = :
Isc

vtu.allsyllabus.com
www.allsyllabus.com

Circuit Theorems j 223

EXAMPLE 3.37
Find the Thevenin and Norton equivalent circuits at the terminals a b for the circuit in
Fig. 3.103.

Figure 3.103

SOLUTION
As a first step in the analysis, let us find Vt :

www.allsyllabus.com

Using the principle of current division,


8 (4 /0 ) 32
Io = =
8 + j 10 j 5 8 + j5
j 320
Vt = Io (j 10) = = 33:92 /58 V
8 + j5
To find Zt , deactivate all the independent sources. This results in a circuit diagram as shown
in Fig. 3.103 (a).

Figure 3.103(a) Figure 3.103(b) Thevenin equivalent circuit

vtu.allsyllabus.com
www.allsyllabus.com

224 j Network Theory

jj
Zt = j 10 (8 j 5) Ω
(j 10)(8 j 5)
=
j 10 + 8 j5
= 10 /26 Ω

The Thevenin equivalent circuit as


viewed from the terminals a b is
as shown in Fig 3.103(b). Performing
source transformation on the Thevenin
equivalent circuit, we get the Norton
equivalent circuit.
Figure : Norton equivalent circuit

Vt 33:92 /58
IN = =
Zt 10 /26
= 3:392 /32 A

EXAMPLE
www.allsyllabus.com
3.38
ZN = Zt = 10 /26 Ω

Find vo using Thevenin’s theorem. Refer to the circuit shown in Fig. 3.104.

Figure 3.104

SOLUTION
Let us convert the circuit given in Fig. 3.104 into a frequency domain equiavalent or phasor circuit
(shown in Fig. 3.105(a)). ! = 1

10 cos (t 45 ) ! 10 / 45 V


5 sin (t + 30 ) = 5 cos (t 60 ) ! 5 / 60 V

L = 1H ! j !L =j 11= 1Ω j

= 1F !
1 1
C
j !C
=
j11 = 1Ω j

vtu.allsyllabus.com
www.allsyllabus.com

Circuit Theorems j 225

Figure 3.105(a)

Disconnecting the capicator from the original circuit, we get the circuit shown in
Fig. 3.105(b). This circuit is used for finding Vt .

www.allsyllabus.com
Figure 3.105(b)
KCL at node a :

Vt 10 / 45 Vt 5 / 60
+ =0
3 j1
Solving; Vt = 4:97 / 40:54

To find Zt deactivate all the independent sources Figure 3.105(c)


in Fig. 3.105(b). This results in a network as
shown in Fig. 3.105(c) :
jj
Zt = Zab = 3Ω j 1 Ω
j3 3
= = (1 + j 3) Ω
3+j 10
The Thevenin equivalent circuit along with the capicator
is as shown in Fig 3.105(d).
Vt
Vo = ( j 1)
Zt j 1
4:97 / 40:54
= ( j 1)
0:3(1 + j 3) j 1
= 15:73 /247:9 V
Hence; vo = 15:73 cos (t + 247:9 ) V
Figure 3.105(d) Thevenin equivalent circuit

vtu.allsyllabus.com
www.allsyllabus.com

226 j Network Theory

EXAMPLE 3.39
Find the Thevenin equivalent circuit of the circuit shown in Fig. 3.106.

Figure 3.106

SOLUTION
Since terminals a b are open,
www.allsyllabus.com
 Va = Is 10
= 20 /0 V

Applying KVL clockwise for the mesh on the right hand side of the circuit, we get

3Va + 0 (j 10) + Voc Va = 0

Voc = 4Va
= 80 /0 V

Let us transform the current source with 10 Ω parallel resistance to a voltage source with 10 Ω
series resistance as shown in figure below :

To find Zt , the independent voltage source is deactivated and a current source of I A is


connected at the terminals as shown below :

vtu.allsyllabus.com
www.allsyllabus.com

Circuit Theorems j 227

Applying KVL clockwise we get,

V0a 3V0a j 10I + Vo = 0


) 4Va0
j 10I + Vo = 0
Since V0a = 10I
we get 40I = Vo
j 10I
Vo
Hence; Zt = = 40 + j 10Ω
I

in Fig 3.106(a) :
www.allsyllabus.com
Hence the Thevenin equivalent circuit is as shown
Figure 3.106(a)

EXAMPLE 3.40
Find the Thevenin and Norton equivalent circuits for the circuit shown in Fig. 3.107.

Figure 3.107

SOLUTION
The phasor equivalent circuit of Fig. 3.107 is shown in Fig. 3.108.
KCL at node a :
Voc 2Voc Voc
10 + =0
j 10 j5

) Voc = j
100 100
3
=
3
/ 90 V

vtu.allsyllabus.com
www.allsyllabus.com

228 j Network Theory

Figure 3.108

To find Isc , short the terminals a b of Fig. 3.108 as in Fig. 3.108(a).

www.allsyllabus.com
Figure 3.108 (a) Figure 3.108 (b)

Since Voc = 0, the above circuit takes the form shown in Fig 3.108 (b).
Isc = 10 /0 A
100
Voc / 90 10
Hence; Zt = = 3 = / 90 Ω
Isc 10 /0
3
The Thevenin equivalent and the Norton equivalent circuits are as shown below.

Figure Thevenin equivalent Figure Norton equivalent

EXAMPLE 3.41
Find the Thevenin and Norton equivalent circuits in frequency domain for the network shown in
Fig. 3.109.

vtu.allsyllabus.com
www.allsyllabus.com

Circuit Theorems j 229

Figure 3.109

SOLUTION
Let us find Vt = Vab using superpostion theorem.
(i) Vab due to 100 /0

www.allsyllabus.com

100 /0 100


I1 = = A
j 300 + j 100 j 200
Vab1 = I1 (j 100)
100
= (j 100) = 50 /0 Volts
j 200

(ii) Vab due to 100 /90

vtu.allsyllabus.com
www.allsyllabus.com

230 j Network Theory

100 /90
I2 =
j 100 j 300
Vab2 = I2 ( j 300)
100 /90
= ( j 300) = j 150 V
j 100 j 300
Hence; Vt = Vab1 + Vab2
= 50 + j 150
= 158:11 /108:43 V

To find Zt , deactivate all the independent sources.

www.allsyllabus.com
Zt = j 100 Ω jj j 300 Ω
j 100( j 300)
= = j 150 Ω
j 100 j 300

Hence the Thevenin equivalent circuit is as shown in Fig. 3.109(a). Performing source trans-
formation on the Thevenin equivalent circuit, we get the Norton equivalent circuit.
Vt 158:11 /108:43
IN = = = 1:054 /18:43 A
Zt 150 /90
ZN = Zt = j 150 Ω

The Norton equivalent circuit is as shown in Fig. 3.109(b).

Figure 3.109(a) Figure 3.109(b)

vtu.allsyllabus.com
www.allsyllabus.com

Circuit Theorems j 231

3.6 Maximum power transfer theorem

We have earlier shown that for a resistive network, maximum power is transferred from a source to
the load, when the load resistance is set equal to the Thevenin resistance with Thevenin equivalent
source. Now we extend this result to the ac circuits.

Figure 3.110 Linear circuit Figure 3.111 Thevenin equivalent circuit

www.allsyllabus.com
In Fig. 3.110, the linear circuit is made up of impedances, independent and dependent sources.
This linear circuit is replaced by its Thevenin equivalent circuit as shown in Fig. 3.111. The load
impedance could be a model of an antenna, a TV, and so forth. In rectangular form, the Thevenin
impedance Zt and the load impedance ZL are

Zt = Rt + jXt
and ZL = RL + jXL

The current through the load is

Vt Vt
I= =
Zt + ZL (Rt + jXt ) + (RL + jXL )

p
The phasors I and Vt are the maximum values. The corresponding RM S values are obtained
by dividing the maximum values by 2. Also, the RM S value of phasor current flowing in the
load must be taken for computing the average power delivered to the load. The average power
delivered to the load is given by

P =
2
jj
1 2
I RL

=
jV j
t
2 RL
2
(3.25)
(Rt + RL ) (Xt + XL )2
2

Our idea is to adjust the load parameters RL and XL so that P is maximum. To do this, we
@P @P
get and equal to zero.
@RL @XL

vtu.allsyllabus.com
www.allsyllabus.com

232 j Network Theory

@P
= h
j j
Vt
2
RL (Xt + XL )
i2
@XL
(Rt + RL )2 + (Xt + XL )2
h i
@P
j j
Vt
2
(Rt + RL )2 + (Xt + XL )2 2RL (Rt + RL )
@RL
= h i2
2 (Rt + RL )2 + (Xt + XL )2

@P
Setting = 0 gives
@XL
XL = Xt (3.26)
@P
and Setting = 0 gives
@RL
q
RL = Rt2 + (Xt + XL )2 (3.27)
Combining equations (3.26) and (3.27), we can conclude that for maximum average power
transfer, ZL must be selected such that XL = Xt and RL = Rt . That is the maximum aver-
age power of a circuit with an impedance Zt that is obtained when ZL is set equal to complex
conjugate of Zt . www.allsyllabus.com
Setting RL = Rt and XL = Xt in equation (3.25), we get the maximum average power as

P =
j jVt
2

8Rt
In a situation where the load is purely real, the condition for maximum power transfer is
obtained by putting XL = 0 in equation (3.27). That is,
q
RL = Rt2 + Xt2 = Zt j j
Hence for maximum average power transfer to a purely resistive load, the load resistance is
equal to the magnitude of Thevenin impedance.

3.6.1 Maximum Power Transfer When Z is Restricted

Maximum average power can be delivered to ZL only if ZL = Zt . There are few situations in
which this is not possible. These situations are described below :

(i) RL and XL may be restricted to a limited range of values. With this restriction,

q XL as close as possible to Xt and then adjust RL as close as possible to


choose
Rt2 + (XL + Xt )2 :

(ii) Magnitude of ZL can be varied but its phase angle cannot be. Under this restriction,
greatest amount of power is transferred to the load when [ZL ] = Zt . j j
Zt is the complex conjugate of Zt .

vtu.allsyllabus.com
www.allsyllabus.com

Circuit Theorems j 233

EXAMPLE 3.42
Find the load impedance that transfers the maximum power to the load and determine the maxi-
mum power quantity obtained for the circuit shown in Fig. 3.112.

Figure 3.112

SOLUTION
We select, ZL = Zt for maximum power transfer.

Hence ZL = 5 + j 6

www.allsyllabus.com
I=
10 /0
5+5
= 1 /0

Hence, the maximum average power transfered to the


load is

P =
1 2
2
jj
I RL
1

= (1)2 5 = 2:5 W
2

EXAMPLE 3.43
Find the load impedance that transfers the maximum average power to the load and determine the
maximum average power transferred to the load ZL shown in Fig. 3.113.

Figure 3.113

vtu.allsyllabus.com
www.allsyllabus.com

234 j Network Theory

SOLUTION
The first step in the analysis is to find the Thevenin equivalent circuit by disconnecting the load
ZL . This leads to a circuit diagram as shown in Fig. 3.114.

Figure 3.114

Hence Vt = Voc = 4 /0 3



= 12 /0 Volts(RMS)

www.allsyllabus.com
To find Zt , let us deactivate all the independent sources of Fig. 3.114. This leads to a circuit
diagram as shown in Fig 3.114 (a):

Zt = 3 + j 4 Ω

Figure 3.114 (a) Figure 3.115

The Thevenin equivalent circuit with ZL is as shown in Fig. 3.115.


For maximum average power transfer to the load, ZL = Zt = 3 j 4.
12 /0
It = = 2 /0 A(RMS)
3 + j4 + 3 j4

Hence, maximum average power delivered to the load is

P = jj
It
2
RL = 4(3) = 12 W
1
It may be noted that the scaling factor is not taken since the phase current is already
2
expressed by its RM S value.

vtu.allsyllabus.com
www.allsyllabus.com

Circuit Theorems j 235

EXAMPLE 3.44
Refer the circuit given in Fig. 3.116. Find the value of RL that will absorb the maximum average
power.

Figure 3.116

SOLUTION

www.allsyllabus.com
Disconnecting the load resistor RL from the original circuit diagram leads to a circuit diagram as
shown in Fig. 3.117.

Figure 3.117

Vt = Voc = I1 (j 20)

=
150 /30 
j 20
(40 j 30 + j 20)
= 72:76 /134 Volts:

To find Zt , let us deactivate all the independent sources present in Fig. 3.117 as shown in
Fig 3.117 (a).

jj
Zt = (40 j 30) j 20
j 20 (40 j 30)
= = (9:412 + j 22:35) Ω
j 20 + 40 j 30

vtu.allsyllabus.com
www.allsyllabus.com

236 j Network Theory

The Value of RL that will absorb the maximum


average power is
p
RL j j
= Zt = (9:412)2 + (22:35)2
= 24:25 Ω

The Thevenin equivalent circuit with RL inserted


is as shown in Fig 3.117 (b).
Maximum average power absorbed by RL is

Pmax =
1
2
jj
It
2
RL
Figure 3.117 (a)

72:76 /134
where It =
(9:412 + j 22:35 + 24:25)
= 1:8 /100:2 A
) 1
Pmax = (1:8)
2
2
24:25 
www.allsyllabus.com
= 39:29 W

Figure 3.117 (b) Thevenin equivalent circuit

EXAMPLE 3.45
For the circuit of Fig. 3.118: (a) what is the value of ZL that will absorb the maximum average
power? (b) what is the value of maximum power?

Figure 3.118

SOLUTION
Disconnecting ZL from the original circuit we get the circuit as shown in Fig. 3.119. The first
step is to find Vt .

vtu.allsyllabus.com
www.allsyllabus.com

Circuit Theorems j 237

Vt = Voc = I1 ( j 10)

 120 /0

= ( j 10)
10 + j 15 j 10

= 107:33 / 116:57 V

The next step is to find Zt . This re-


quires deactivating the independent Figure 3.119
voltage source of Fig. 3.119.

Zt = (10 + j 15) ( jj j 10)

j 10 (10 + j 15)
=
j 10 + 10 + j 15

=8 j 14 Ω

www.allsyllabus.com
The value of ZL for maximum average power absorbed is

Zt = 8 + j 14 Ω

The Thevenin equivalent circuit along with ZL = 8 + j 14 Ω is as shown below:

107:33 / 116:57
It =
8 j 14 + 8 + j 14
107:33
= / 116:57 A
16
Hence; Pmax =
1
jj
2
It R L
2
 
=
1 107:33 2
2 16
8 
= 180 Walts

vtu.allsyllabus.com
www.allsyllabus.com

238 j Network Theory

EXAMPLE 3.46
(a) For the circuit shown in Fig. 3.120, what is the value of ZL that results in maximum average
power that will be transferred to ZL ? What is the maximum power ?
(b) Assume that the load resistance can be varied between 0 and 4000 Ω and the capacitive
reactance of the load can be varied between 0 and 2000 Ω. What settings of RL and XC
transfer the most average power to the load ? What is the maximum average power that can
be transferred under these conditions?

www.allsyllabus.com Figure 3.120

SOLUTION
(a) If there are no constraints on RL and XL , the load indepedance ZL = Zt = (3000 j 4000) Ω.
Since the voltage source is given in terms of its RM S value, the average maximum power
delivered to the load is

Pmax = Itjj 2
RL
10 /0
where It =
3000 + j 4000 + 3000 j 4000
10
=

2 3000
A

) 2
jj
Pmax = It RL

=
100

4 (3000)2
3000 
= 8:33 mW

(b) Since RL and XC are restricted, we firstqset XC as close to 4000 Ω as possible; hence
2
XC = 2000 Ω. Next we set RL as close to Rt2 + (XC + XL ) as possible.
q
Thus, RL = 30002 + ( 2000 + 4000)2 = 3605:55 Ω
Since RL can be varied between 0 to 4000 Ω, we can set RL to 3605:55 Ω. Hence ZL is
adjusted to a value
ZL = 3605:55 j 2000 Ω:

vtu.allsyllabus.com
www.allsyllabus.com

Circuit Theorems j 239

10 /0
It =
3000 + j 4000 + 3605:55 j 2000
= 1:4489 / 16:85 mA

The maximum average power delivered


to the load is

Pmax jj
= It 2
RL

= 1:4489  10 3 2
 3605 55
:

= 7:57 mW
Note that this is less than the power that can be delivered if there are no constraints on RL
and XL .

EXAMPLE 3.47
A load impedance having a constant phase angle of 45 is connected across the load terminals
a and b in the circuit shown in Fig. 3.121. The magnitude of ZL is varied until the average power

www.allsyllabus.com
delivered, which is the maximum possible under the given restriction.
(a) Specify ZL in rectangular form.
(b) Calculate the maximum average power delivered under this condition.

Figure 3.121

SOLUTION
Since the phase angle of ZL is fixed at 45 , for maximum power transfer to ZL it is mandatory
that
jZ j = pjZ j
L t
= (3000)2 + (4000)2
= 5000 Ω:
Hence; j j
ZL = ZL / 45
=
5000
2
j p
5000
2
p

vtu.allsyllabus.com
www.allsyllabus.com

240 j Network Theory

10 /0
It =
(3000 + 3535:53) + j (4000 3535:53)
= 1:526 / 4:07 mA
Pmax jj
= It 2
RL

= 1:526  10 3 2
 3535 53:

= 8:23 mW

This power is the maximum average power that can be delivered by this circuit to a load
impedance whose angle is constant at 45 . Again this quantity is less than the maximum
power that could have been delivered if there is no restriction on ZL . In example 3.46 part (a),
we have shown that the maximum power that can be delivered without any restrictions on ZL
is 8.33 mW.

3.7 Reciprocity theorem


www.allsyllabus.com
The reciprocity theorem states that in a linear bilateral single source circuit, the ratio of exci-
tation to response is constant when the positions of excitation and response are interchanged.

Conditions to be met for the application of reciprocity theorem :

(i) The circuit must have a single source.


(ii) Initial conditions are assumed to be absent in the circuit.
(iii) Dependent sources are excluded even if they are linear.
(iv) When the positions of source and response are interchanged, their directions should be marked
same as in the original circuit.

EXAMPLE 3.48
Find the current in 2 Ω resistor and hence verify reciprocity theorem.

Figure 3.122

vtu.allsyllabus.com
www.allsyllabus.com

Circuit Theorems j 241

SOLUTION
The circuit is redrawn with markings as shown in Fig 3.123 (a).

Figure 3.123 (a)


1 1 1
Then; R1 = (8 +2 ) = 1:6Ω
R2 = 1:6 + 4 = 5:6Ω
= (5:6 1 + 4 1 ) 1 = 2:3333Ω
www.allsyllabus.com
R3

Current supplied by the source =


20
4 + 2:3333
= 3:16 A

Current in branch ab = Iab = 3:16  4


4 + 4 + 1: 6
= 1:32 A

Current in 2Ω; I1 = 1:32  8


10
= 1:05 A

Verification using reciprocity theorem


The circuit is redrawn by interchanging the position of excitation and response as shown in
Fig 3.123 (b).

Figure 3.123 (b)


Solving the equivalent resistances,
R4 = 2Ω; R5 = 6Ω; R6 = 3:4286Ω
Now the current supplied by the source
20
= = 3:6842A
3:4286 + 2

vtu.allsyllabus.com
www.allsyllabus.com

242 j Network Theory

Therefore,
Icd = 3:6842  8 +8 6 = 2 1053A
:

2:1053
I2 = = 1:05A
2
As I1 = I2 = 1:05 A, reciprocity theorem is verified.

EXAMPLE 3.49
In the circuit shown in Fig. 3.124, find the current through 1:375 Ω resistor and hence verify
reciprocity theorem.

www.allsyllabus.com
Figure 3.124

SOLUTION

Figure 3.125
KVL clockwise for mesh 1 :
6:375I1 2I2 3I3 = 0
KVL clockwise for mesh 2 :
2I1 + 14I2 10I3 = 0
KVL clockwise for mesh 3 :
3I1 10I2 + 14I3 = 10

vtu.allsyllabus.com
www.allsyllabus.com

Circuit Theorems j 243

Putting the above three mesh equations in matrix form, we get


2 32 3 2 3
6:375 2 3 I1 0
4 2 14 10 54 I2 5=4 0 5
3 10 14 I3 10
Using Cramer’s rule, we get
I1 = 2A
Negative sign indicates that the assumed direction of current flow should have been the other way.
Verification using reciprocity theorem :
The circuit is redrawn by interchanging the positions of excitation and response. The new circuit
is shown in Fig. 3.126.

www.allsyllabus.com
Figure 3.126
The mesh equations in matrix form for the circuit shown in Fig. 3.126 is
2 32 0
3 2 3
6:375 2 3 I1 10
4 2 14 10 54 0
I2 5=4 0 5
0
3 10 14 I3 0
Using Cramer’s rule, we get
0
I3 = 2A
Since I1 = I30 = 2 A, the reciprocity theorem is verified.

EXAMPLE 3.50
Find the current Ix in the j 2 Ω impedance and hence verify reciprocity theorem.

Figure 3.127

vtu.allsyllabus.com
www.allsyllabus.com

244 j Network Theory

SOLUTION
With reference to the Fig. 3.127, the current through j 2 Ω impepance is found using series parallel
reduction techniques.
Total impedance of the circuit is
jj
ZT = (2 + j 3) + ( j 5) (3 + j 2)
( j 5)(3 + j 2)
= 2 + j3 +
j5 + 3 + j2
= 6:537 /19:36 Ω
The total current in the network is
36 /0
IT =
6:537 /19:36
= 5:507 / 19:36 A
Using the principle of current division, we find that
IT ( j 5)
Ix =
j5 + 3 + j2

www.allsyllabus.com
Verification of reciprocity theorem :
= 6:49 / 64:36 A

The circuit is redrawn by changing the positions of excitation and response. This circuit is shown
in Fig. 3.128.
Total impedance of the circuit shown in
Fig. 3.128 is

jj
Z0T = (3 + j 2) + (2 + j 3) ( j 5)
(2 + j 3) ( j 5)
= (3 + j 2) +
2 + j3 j5
= 9:804 /19:36 Ω

The total current in the circuit is


36 /0 Figure 3.128
I0T = 0
= 3:672 / 19:36 A
ZT

Using the principle of current division,


I0T ( j 5)
Iy = = 6:49 / 64:36 A
j5 + 2 + j3

It is found that Ix = Iy , thus verifying the reciprocity theorem.

EXAMPLE 3.51
Refer the circuit shown in Fig. 3.129. Find current through the ammeter, and hence verify reci-
procity theorem.

vtu.allsyllabus.com
www.allsyllabus.com

Circuit Theorems j 245

Figure 3.129

SOLUTION

To find the current through the ammeter :


By inspection the loop equations for the circuit in Fig. 3.130 can

2
16
www.allsyllabus.com
be written in the matrix form as
32 3 2 3
1 10 I1 0
4 1 26 20 54 I2 5=4 0 5
10 20 30 I3 50

Using Cramer’s rule, we get

I1 = 4:6 A
I2 = 5:4 A

Hence current through the ammeter = I2 I1 = 5:4 4:6 = 0:8A. Figure 3.130

Verification of reciprocity theorem:


The circuit is redrawn by interchanging the positions of
excitation and response as shown in Fig. 3.131.
By inspection the loop equations for the circuit can be
written in matrix form as
2 32 0
3 2 3
15 0 10 I1 50
4 0 25 20 54 0
I2 5=4 50 5
0
10 20 31 I3 0

Using Cramer’s rule we get


0
I3 = 0:8 A
Figure 3.131

vtu.allsyllabus.com
www.allsyllabus.com

246 j Network Theory

Hence, current through the Ammeter = 0.8 A.


It is found from both the cases that the response is same. Hence the reciprocity theorem is
verified.
EXAMPLE 3.52
Find current through 5 ohm resistor shown in Fig. 3.132 and hence verify reciprocity theorem.

SOLUTION
www.allsyllabus.com Figure 3.132

By inspection, we can write


2 32 3 2 3
12 0 2 I1 20
4 0 2 + j 10 2 54 I2 5=4 20 5
2 2 9 I3 0
Using Cramer’s rule, we get
I3 = 0:5376 / 126:25 A
Hence, current through 5 ohm resistor = 0:5376 / 126:25 A
Verification of reciprocity theorem:
The original circuit is redrawn by interchanging the excitation and response as shown in Fig.
3.133.

Figure 3.133

vtu.allsyllabus.com
www.allsyllabus.com

Circuit Theorems j 247

Putting the three equations in matrix form, we get


2 32 3 2 3
12 0 2 I01 0
4 0 2 + j 10 2 5 64 I02
75 = 4 0 5
2 2 9 I03 20

Using Cramer’s rule, we get

I01 = 0:3876 / 2:35 A


I02 = 0:456 / 78:9 A
Hence; I02 I01 = 0:3179 j 0:4335

= 0:5376 / 126:25 A

The response in both cases remains the same. Thus verifying reciprocity theorem.

3.8 Millman’s theorem

www.allsyllabus.com
It is possible to combine number of voltage sources or current sources into a single equiva-
lent voltage or current source using Millman’s theorem. Hence, this theorem is quite useful in
calculating the total current supplied to the load in a generating station by a number of generators
connected in parallel across a busbar.
Millman’s theorem states that if n number of generators having generated emfs E1 , E2 ; En 
and internal impedances Z1 ; Z2 ; 
Zn are connected in parallel, then the emfs and impedances
can be combined to give a single equivalent emf of E with an internal impedance of equivalent
value Z.
E1 Y1 + E2 Y2 + : : : + En Yn
where E=
Y1 + Y2 + : : : + Yn
1
and Z=
Y1 + Y2 + : : : + Yn
where Y1 ; Y2 
Yn are the admittances corresponding to the internal impedances Z1 ; Z2  Z n
and are given by
1
Y1 =
Z1
1
Y2 =
Z2
..
.
1
Yn =
Zn
Fig. 3.134 shows a number of generators having emfs E1 ; E2 
En connected in parallel
across the terminals x and y . Also, Z1 ; Z2 
Zn are the respective internal impedances of the
generators.

vtu.allsyllabus.com
www.allsyllabus.com

248 j Network Theory

Figure 3.134

The Thevenin equivalent circuit of Fig. 3.134 using Millman’s theorem is shown in Fig. 3.135.
The nodal equation at x gives

E1 E E2 E
Z1
+
Z2
+ 
+
En E
Zn
=0
E E  1 1 
) Z1
1

+
Z2
2
+ 
+
En
Zn
=E + + +
1

 Z1 1 Z2 Zn

) www.allsyllabus.com

E1 Y1 + E2 Y2 + + En Yn = E
Z
Figure 3.135
where Z = Equivalent internal impedance.
or  + E Y ] = EY
[E1 Y1 + E2 Y2 + n n
E Y + E Y +  + E Y
) E=
1 1
Y
2 2 n n

where Y = Y + Y +  + Y
1 2 n
1 1
Y + Y +  + Y
and Z= =
Y 1 2 n

EXAMPLE 3.53
Refer the circuit shown in Fig. 3.136. Find the current through 10 Ω resistor using Millman’s
theorem.

Figure 3.136

vtu.allsyllabus.com
www.allsyllabus.com

Circuit Theorems j 249

SOLUTION
Using Millman’s theorem, the circuit shown in Fig. 3.136 is replaced by its Thevenin equivalent
circuit across the terminals P Q as shown in Fig. 3.137.

E1 Y1 + E2 Y2 E3 Y3
E=
Y1 + Y2 + Y3
1 1 1
22 + 48 12
5 12 4
=
1 1 1
+ +
5 12 4
= 10:13 Volts
1
R=
Y1 + Y2 + Y3
1 Figure 3.137
=
0:2 + 0:083 + 0:25
= 1:88 Ω

Hence;
www.allsyllabus.com IL =
E
= 0:853 A
R + 10

EXAMPLE 3.54
Find the current through (10 j 3)Ω using Millman’s theorem. Refer Fig. 3.138.

Figure 3.138

SOLUTION
The circuit shown in Fig. 3.138 is replaced by its Thevenin equivalent circuit as seen from the
terminals, A and B using Millman’s theorem. Fig. 3.139 shows the Thevenin equivalent circuit
along with ZL = 10 j 3 Ω:

vtu.allsyllabus.com
www.allsyllabus.com

250 j Network Theory

Figure 3.139

E1 Y1 + E2 Y2 E3 Y3
E=
Y1 + Y2 + Y3
1 1 1
  
100 /0 + 90 /45 + 80 /30
5 10 20
=
www.allsyllabus.com 1
+
1
+
5 10 20
1

= 88:49 /15:66 V
1 1
Z=R= = 1 1 1 = 2:86 Ω
Y1 + Y2 + Y3 5 + 10 + 20

E 88:49 /15:66
I= = = 6:7 /28:79 A
Z + ZL 2:86 + 10 j 3
Alternately,
E1 Y1 + E2 Y2 + E3 Y3 + E4 Y4
E=
Y1 + Y2 + Y3 + Y4
 
=
100 5 1 
+ 90 45 10 1 + 80 30  20 1

5 1 + 10 1 + 20 1 + (10 j 3) 1

= 70 /12 V
70 /12
Therefore; I =
10 j 3
= 6:7 /28:8 A

EXAMPLE 3.55
Refer the circuit shown in Fig. 3.140. Use Millman’s theorem to find the current through (5+j 5) Ω
impedance.

vtu.allsyllabus.com
www.allsyllabus.com

Circuit Theorems j 251

Figure 3.140
SOLUTION
The original circuit is redrawn after performing source transformation of 5 A in parallel with 4 Ω
resistor into an equivalent voltage source and is shown in Fig. 3.141.

www.allsyllabus.com

Figure 3.141

Treating the branch 5 + j 5Ω as a branch with Es = 0V ,


E1 Y1 + E2 Y2 + E3 Y3 + E4 Y4
EP Q =
Y1 + Y2 + Y3 + Y4

= 1
 
4 2 1 + 8 3 1 + 20 4 1 
2 + 3 1 + 4 1 + (5 j 5) 1
= 8:14 /4:83 V
Therefore current in (5 + j 5)Ω is
8:14 /4:83
I= = 1:15 / 40:2 A
5 + j5
Alternately
EP Q with (5 + j 5) open
E1 Y1 + E2 Y2 + E3 Y3
EP Q =
Y1 + Y2 + Y3

=
 
4 2 1 + 8 3 1 + 20 4 1

2 1+3 1+4 1
= 8:9231V

vtu.allsyllabus.com
www.allsyllabus.com

252 j Network Theory

Equivalent resistance R = (2 1 + 3 1 +4 1) 1 = 0:9231Ω


Therefore current in (5 + j 5)Ω is
8:9231
I = = 1:15 / 40:2 A
0:9231 + 5 + j 5

EXAMPLE 3.56
Find the current through 2 Ω resistor using Millman’s theorem. Refer the circuit shown
in Fig. 3.142.

www.allsyllabus.com
Figure 3.142
SOLUTION
The Thevenin equivalent circuit using Millman’s theorem for the given problem is as shown in
Fig. 3.142(a).
E1 Y1 + E2 Y2
where E=
Y1 + Y2
 1
 1
10 /10 + 25 /90
3 + j4 5
=
1 1
+
3 + j4 5
= 10:06 /97:12 V
1 1
Z= =
Y1 + Y2 1 1
+
3 + j4 5
= 2:8 /26:56 Ω
E 10:06 /97:12
Hence; IL = =
Z + 2 2:8 /26:56 + 2
= 2:15 /81:63 A
Figure 3.142(a)

vtu.allsyllabus.com
www.allsyllabus.com

Circuit Theorems j 253

Reinforcement problems

R.P 3.1
Find the current in 2 Ω resistor connected between A and B by using superposition theorem.

Figure R.P. 3.1

SOLUTION

www.allsyllabus.com
Fig. R.P. 3.1(a), shows the circuit with 2V-source acting alone (4V-source is shorted).
Resistance as viewed from 2V-source is 2 + R1 Ω,

3  2 
where R1 = + 1 12
5

=
(1:2 + 1) 12  = 1:8592 Ω
14:2
2
Hence; Ia = = 0:5182 A
2 + 1:8592
Then; Ib = Ia  12
12 + 1 + 1:2
= 0:438 A

∴ I1 = 0:438
3
5
= 0:2628 A Figure R.P. 3.1(a)

With 4V-source acting alone, the circuit is as shown in Fig. R.P. 3.1(b).

Figure R.P.3.1(b)

vtu.allsyllabus.com
www.allsyllabus.com

254 j Network Theory

The resistance as seen by 4V-source is 3 + R2 where


 2  12 
R2 = +1 2
14
=

2:7143 2
= 1:1551 Ω
4:7143
4
Hence; Ib = = 0:9635 A

3 + 1:1551
Ib 2:7143
Thus; I2 = = 0:555 A
4:7143
Finally, applying the principle of superposition,

we get, IAB = I1 + I2
= 0:2628 + 0:555
= 0:818 A
R.P 3.2

www.allsyllabus.com
For the network shown in Fig. R.P. 3.2, apply superposition theorem and find the current I.

Figure R.P. 3.2

SOLUTION
Open the 5A-current source and retain the voltage source. The resulting network is as shown in
Fig. R.P. 3.2(a).

Figure R.P. 3.2(a)

vtu.allsyllabus.com
www.allsyllabus.com

Circuit Theorems j 255

The impedance as seen from the voltage source is


(8 + j 10) ( j 2)
Z = (4 j 2) + = 6:01 / 45 Ω
8 + j8
j 20
Hence; Ia =
= 3:328 /135 A
Z
Next, short the voltage source and retain the current source. The resulting network is as shown
in Fig. R.P. 3.2 (b).
Here, I3 = 5A. Applying KVL for mesh 1 and mesh 2, we
get
8I1 + (I1 5) j 10 + (I1 I2 ) ( j 2) =0
and (I2 I1 ) ( j 2) + (I2 5) ( j 2) + 4I2 = 0

Simplifying, we get

(8 + j 8)I1 + j 2I2 = j 50
and j 2I1 + (4 j 4)I2 = j 10

Solving, we get www.allsyllabus.com


8 + j8 j 50
j2 j 10
Ib = I 2 =
8 + j8 j2
j2 4 j4
= 2:897 / 23:96 A
Figure R.P. 3.2(b)
Since, Ia and Ib are flowing in opposite directions, we
have
I = Ia Ib = 6:1121 /144:78 A

R.P 3.3
Apply superposition theorem and find the voltage across 1 Ω resistor. Refer the circuit shown in
Fig. R.P. 3.3. Take v1 (t) = 5 cos (t + 10 ) and i2 (t) = 3 sin 2t A.

Figure R.P. 3.3

vtu.allsyllabus.com
www.allsyllabus.com

256 j Network Theory

SOLUTION
To begin with let us assume v1 (t) alone is acting. Accordingly, short 10V - source and open i2 (t).
The resulting phasor network is shown in Fig. R.P. 3.3(a).
! = 1rad=sec

5 cos (t + 10 ) ! 5 /10 
V
L1 = 1H ! j !L1 = j1 Ω
= 1F !
1
C1 = j1 Ω
j !C1

= H!
1 1
L2 j !L2 =j Ω
2 2
= F!
1 1 Figure R.P. 3.3(a)
C2 = j2 Ω
2 j !C2

∴ Va = 5 /10 V
) va (t) = 5 cos [t + 10 ]

Let us next assume that i2 (t) alone is acting. The resulting network is shown
www.allsyllabus.com
in Fig. R.P. 3.3(b).
! = 2 rad=sec
3 sin 2t! 3 /0 
A
= 1F !
1 1
C1 Ω= j
j !C1 2
L1 = 1H ! j !L1 = j 2 Ω

= F!
1 1
C2 = j1 Ω
2 j !C2

= H!
1 Figure R.P. 3.3(b)
L2 j !L2 = j1 Ω
2

Vb = 3 /0  1 +1 51 5 = 2 5 /33 7
j :
j :
: :

A
) vb (t) = 2:5 sin [2t + 33:7 ] A

Finally with 10V-source acting alone, the network is as shown in Fig. R.P. 3.3(c). Since
! = 0, inductors are shorted and capacitors are opened.
Hence, Vc = 10 V
Applying principle of superposition, we
get.

v2 (t) = va (t) = vb (t) + Vc


= 5 cos (t + 10 ) + 2:5 sin (2t + 33:7 ) + 10Volts
Figure R.P. 3.3(c)

vtu.allsyllabus.com
www.allsyllabus.com

Circuit Theorems j 257

R.P 3.4
Calculate the current through the galvanometer for the Kelvin double bridge shown in Fig. R.P.
3.4. Use Thevenin’s theorem. Take the resistance of the galvanometer as 30 Ω.

Figure R.P. 3.4

SOLUTION
www.allsyllabus.com
With G being open, the resulting network is as shown in Fig. R.P. 3.4(a).

Figure 3.4(a)

VA  100 = 450
= I1
10
 100 = 209 V
10 5 =01
45  5
I2
I2 = = 1 66 := ; IB : I2
45 + 5
15+
:

= 05+  10
50
Hence; VB I2 : IB

= 2:5 V
20 5
Thus; VAB = Vt = VA VB = 2:5 = Volts
9 18

vtu.allsyllabus.com
www.allsyllabus.com

258 j Network Theory

To find Rt , short circuit the voltage source. The resulting network is as shown in Fig. R.P. 3.4(b).

Figure R.P. 3.4 (b)

Transforming the Δ between B , E and F into an equivalent Y , we get

RB =
35  10 = 7 Ω; RE =

35 5
= 3:5 Ω; RF =
5  10 = 1 Ω
50 50 50

The reduced network after transformation is as shown in Fig. R.P. 3.4(c).

www.allsyllabus.com

Figure R.P. 3.4(c)

Hence; RAB = Rt =

350 100 4:5 1:5
+ +7

450 6
= 85:903 Ω

The Thevenin’s equivalent circuit as seen from A


and B with 30 Ω connected between A and B is
as shown in Fig. R.P. 3.4(d).
5
IG =
18 = 2:4mA
85:903 + 30
Negative sign implies that the current flows from
B to A.
Figure R.P. 3.4(d)

R.P 3.5
Find Is and R so that the networks N1 and N2 shown in Fig. R.P. 3.5 are equivalent.

vtu.allsyllabus.com
www.allsyllabus.com

Circuit Theorems j 259

Figure R.P. 3.5


SOLUTION
Transforming the current source in N1 into an equivalent voltage source, we get N3 as shown in
Fig. R.P. 3.5(a).
From N3 , we can write, V I R = IS R (3.28)
From N2 we can write, I = 10Ia
Also from N2 , V 3=

2Ia

) 3= 2
10
V
I

www.allsyllabus.com
) 3=
5
V
I

) 5
=3 (3.29)
V
I

For equivalence of N1 and N2 , it is requirred that equations


(3.28) and (3.29) must be same. Comparing these equations, we
get
I
IR = and IS R =3
5
3
R = 0:2 Ω and IS = = 15A
0:2 Figure R.P. 3.5(a)
R.P 3.6
Obtain the Norton’s equivalent of the network shown in Fig. R.P. 3.6.

Figure R.P. 3.6

vtu.allsyllabus.com
www.allsyllabus.com

260 j Network Theory

SOLUTION
Terminals a and b are shorted. This results in a network as shown in Fig. R.P. 3.6(a)

Figure R.P. 3.6(a)

The mesh equations are

(i) 9I1 + 0I2 6I3 = 30 (3.30)


(ii)
(iii) www.allsyllabus.com 0I1 + 25I2 + 15I3 = 30
6I1 + 15I2 + 23I3 = 4VX = 4 (10I2 )
(3.31)

) 6I1 25I2 + 23I3 = 0 (3.32)

Solving equations (3.30), (3.31) and (3.32), we get


IN = Isc = I3 = 1:4706A

With terminals ab open, I3 = 0. The corresponding equations are

9I1 = 30 and
25I2 = 50
30 30
Hence; I1 = A and I2 = A
9 25
Then; VX = 10I2 = 10
30
25

= 12 V
Hence; Vt = Voc = 15I2 6I1 4VX
= 50 V
Voc 50
Thus; Rt = = = 34 Ω
Isc 1:4706
Hence, Norton’s equivalent circuit is as shown in Fig. R.P. 3.6(b).

Figure R.P. 3.6(b)

vtu.allsyllabus.com
www.allsyllabus.com

Circuit Theorems j 261

R.P 3.7
For the network shown in Fig. R.P. 3.7, find the Thevenin’s equivalent to show that

V1
Vt = (1 + a + b ab)
2
3 b
and Zt =
2

SOLUTION
www.allsyllabus.com Figure R.P. 3.7

V1 aV1
With xy open, I1 =
2
Hence,

Voc = Vt = aV1 + I1 + bI1


V1 aV1
V aV1

1
= aV1 + +b
2 2
V1
= [1 + a + b ab]
2
With xy shorted, the resulting network is
as shown in Fig. R.P. 3.7(a). Figure R.P. 3.7(a)

Applying KVL equations, we get

(i) I1 + (I1 I2 ) = V1 aV1

) 2I1 I2 = V1 aV1 (3.33)


(ii) (I2 I1 ) + I2 = aV1 + bI1
) (1 + b) I1 + 2I2 = aV1 (3.34)

Solving equations (3.33) and (3.34), we get

V1 (1 + a + b ab)
Isc = I2 =
3 b

vtu.allsyllabus.com
www.allsyllabus.com

262 j Network Theory

Voc V1 (1 + a + b ab)
Hence; Zt = = (3 b)
Isc 2 V1 (1 + a + b ab)
3 b
=
2

R.P 3.8
Use Norton’s theorem to determine I in the network shown in Fig. R.P. 3.8. Resistance Values
are in ohms.

www.allsyllabus.com

Figure R.P. 3.8

SOLUTION
Let IAE = x and IEF = y . Then by applying KCL at various junctions, the branch currents are
marked as shown in Fig. R.P. 3.8(a). Isc = 125 x = IAB on shorting A and B .
Applying KVL to the loop ABC F EA, we get

0:04x + 0:01y + 0:02 (y 20) + 0:03 (x 105) = 0


) 0:07x + 0:03y = 3:55 (3.35)

Applying KVL to the loop EDC EF , we get

(x y 30) 0:03 + (x y 55) 0:02 (y 20) 0:02 0:01y = 0


) 0:05x 0:08y = 1:6 (3.36)

vtu.allsyllabus.com
www.allsyllabus.com

Circuit Theorems j 263

www.allsyllabus.com Figure R.P. 3.8(a)

Solving equations (3.35) and (3.36), we get


x = 46:76 A
Hence; Isc = IN = 120 x

= 78:24 A
The circuit to calculate Rt is as shown in Fig. R.P. 3.8(b). All injected currents have been
opened.

Rt = 0:03 + 0:04 +

0:03 0:05
0:08
= 0:08875 Ω

Figure R.P. 3.8(b) Figure R.P. 3.8(c)

vtu.allsyllabus.com
www.allsyllabus.com

264 j Network Theory

The Norton’s equivalent network is as shown in Fig. R.P. 3.8(c).

I = 78:24  0 08875
0 08875
: + 0 04
:
:
= 53:9A

R.P 3.9
For the circuit shown in Fig. R.P. 3.9, find R such that the maximum power delivered to the load
is 3 mW.

www.allsyllabus.com
Figure R.P. 3.9

SOLUTION
For a resistive network, the maximum power delivered to the load is
2
Vt
Pmax =
4Rt
The network with RL removed is as shown in Fig.
R.P. 3.9(a).
Let the opent circuit voltage between the termi-
nals a and b be Vt .
Then, applying KCL at node a, we get
Figure R.P. 3.9(a)
Vt 1 Vt 2 Vt 3
+ + =0
R R R

Simplifying we get Vt = 2 Volts


With all voltage sources shorted, the resistance, Rt as viewed from the terminals, a and b is
found as follows:
1 1 1 1 3
= + + =
Rt R R R R

) Rt =
R
3
Ω

vtu.allsyllabus.com
www.allsyllabus.com

Circuit Theorems | 265

22 3
Hence, Pmax = = = 3 × 10−3
4× 3 R R
⇒ R = 1 kΩ
R.P 3.10
Refer Fig. R.P. 3.10, find X1 and X2 interms of R1 and R2 to give maximum power dissipation
in R2 .

Figure R.P. 3.10

SOLUTION www.allsyllabus.com
The circuit for finding Zt is as shown in Figure R.P. 3.10(a).

R1 (jX1 )
Zt =
R1 + jX1
R1 X12 + jR12 X1
=
R12 + X12
Figure R.P. 3.10(a)
For maximum power transfer,
ZL = Z∗t
R1 X12 R12 X1
⇒ R2 + jX2 = − j
R12 + X12 R12 + X12
R1 X12
Hence, R2 =
R12 + X12

R2
⇒ X1 = ±R1 (3.37)
R1 − R2
R12 X1
X2 = − (3.38)
R12 + X12
Substituting equation (3.37) in equation (3.38) and simplifying, we get

X2 = R2 (R1 − R2 )

vtu.allsyllabus.com
www.allsyllabus.com

266 j Network Theory

Exercise Problems
E.P 3.1
Find ix for the circuit shown in Fig. E.P. 3.1 by using principle of superposition.

Figure E.P. 3.1


1
Ans : ix = A
4
E.P 3.2

www.allsyllabus.com
Find the current through branch P Q using superposition theorem.

Figure E.P. 3.2


Ans : 1.0625 A
E.P 3.3
Find the current through 15 ohm resistor using superposition theorem.

Figure E.P. 3.3


Ans : 0.3826 A

vtu.allsyllabus.com
www.allsyllabus.com

Circuit Theorems j 267

E.P 3.4
Find the current through 3 + j 4 Ω using superposition theorem.

Figure E.P. 3.4

Ans : 8.3 /85.3 A

E.P 3.5
Find the current through Ix using superposition theorem.
www.allsyllabus.com

Figure E.P. 3.5

Ans : 3.07 / 163.12 A

E.P 3.6
Determine the current through 1 Ω resistor using superposition theorem.

Figure E.P. 3.6

Ans : 0.406 A

vtu.allsyllabus.com
www.allsyllabus.com

268 j Network Theory

E.P 3.7
Obtain the Thevenin equivalent circuit at terminals a b of the network shown in Fig. E.P. 3.7.

Figure E.P. 3.7


Ans : Vt = 6.29 V, Rt = 9.43 Ω

E.P 3.8
Find the Thevenin equivalent circuit at terminals x y of the circuit shown in Fig. E.P. 3.8.

www.allsyllabus.com

Figure E.P. 3.8


Ans : Vt = 0.192 / 43.4 V, Zt = 88.7 /11.55 Ω

E.P 3.9
Find the Thevenin equivalent of the network shown in Fig. E.P. 3.9.

Figure E.P. 3.9


Ans : Vt = 17.14 volts, Rt = 4 Ω

vtu.allsyllabus.com
www.allsyllabus.com

Circuit Theorems j 269

E.P 3.10
Find the Thevenin equivalent circuit across a b. Refer Fig. E.P. 3.10.

Figure E.P. 3.10

Ans : Vt = 30 V, Rt = 10 kΩ

E.P 3.11
Find the Thevenin equivalent circuit across a b for the network shown in Fig. E.P. 3.11.

www.allsyllabus.com

Figure E.P. 3.11

Ans : Verify your result with other methods.

E.P 3.12
Find the current through 20 ohm resistor using Norton equivalent.

Figure E.P. 3.12

Ans : IN = 4.36 A, RN = Rt = 8.8 Ω, IL = 1.33 A

vtu.allsyllabus.com
www.allsyllabus.com

270 j Network Theory

E.P 3.13
Find the current in 10 ohm resistor using Norton’s theorem.

Figure E.P. 3.13


100
Ans : IN = 4 A, Rt = RN = Ω, IL = 0.5 A
7
E.P 3.14
Find the Norton equivalent circuit between the terminals a b for the network shown in Fig. E.P. 3.14.
www.allsyllabus.com

Figure E.P. 3.14


Ans : IN = 4.98310 / 5.71 A, Zt = ZN = 3.6 /23.1 Ω

E.P 3.15
Determine the Norton equivalent circuit across the terminals P Q for the network shown in
Fig. E.P. 3.15.

Figure E.P. 3.15


Ans : IN = 5 A, RN = Rt = 6 Ω

vtu.allsyllabus.com
www.allsyllabus.com

Circuit Theorems j 271

E.P 3.16
Find the Norton equivalent of the network shown in Fig. E.P. 3.16.

Figure E.P. 3.16

Ans : IN = 8.87 A, RN = Rt = 43.89 Ω

E.P 3.17
Determine the value of RL for maximum power transfer and also find the maximum power trans-
ferred.

www.allsyllabus.com

Figure E.P. 3.17

Ans : RL = 1.92 Ω, Pmax = 4.67 W

E.P 3.18
Calculate the value of ZL for maximum power transfer and also calculate the maximum power.

Figure E.P. 3.18

Ans : ZL = (7.97 + j2.16)Ω, Pmax = 0.36 W

vtu.allsyllabus.com
www.allsyllabus.com

272 j Network Theory

E.P 3.19
Determine the value of RL for maximum power transfer and also calculate the value of maximum
power.

Figure E.P. 3.19

Ans : RL = 5.44 Ω, Pmax = 2.94 W

E.P 3.20
Determine the value of ZL for maximum power transfer. What is the value of maximum power?

www.allsyllabus.com

Figure E.P. 3.20

Ans : ZL = 4.23 + j1.15 Ω, Pmax = 5.68 Watts

E.P 3.21
Obtain the Norton equivalent across x y.

Figure E.P. 3.21

Ans : IN = ISC = 7.35A, Rt = RN = 1.52 Ω

E.P 3.22
Find the Norton equivalent circuit at terminals a b of the network shown in Fig. E.P. 3.22.

vtu.allsyllabus.com
www.allsyllabus.com

Circuit Theorems j 273

Figure E.P. 3.22

Ans : IN = 1.05 /251.6 A, Zt = ZN = 10.6 /45 Ω

E.P 3.23
Find the Norton equivalent across the terminals X Y of the network shown in Fig. E.P. 3.23.

www.allsyllabus.com

Figure E.P. 3.23

Ans : IN = 7A, Zt = 8.19 / 55 Ω

E.P 3.24
Determine the current through 10 ohm resistor using Norton’s theorem.

Figure E.P. 3.24

Ans : 0.15A

vtu.allsyllabus.com
www.allsyllabus.com

274 j Network Theory

E.P 3.25
Determine the current I using Norton’s theorem.

Figure E.P. 3.25

Ans : Verify your result with other methods.

E.P 3.26
Find Vx in the circuit shown in Fig. E.P. 3.26 and hence verify reciprocity theorem.

www.allsyllabus.com

Figure E.P. 3.26

Ans : Vx = 9.28 /21.81 V

E.P 3.27
Find Vx in the circuit shown in Fig. E.P. 3.27 and hence verify reciprocity theorem.

Figure E.P. 3.27

Ans : Vx = 10.23 Volts

vtu.allsyllabus.com
www.allsyllabus.com

Circuit Theorems j 275

E.P 3.28
Find the current ix in the bridge circuit and hence verify reciprocity theorem.

Figure E.P. 3.28


Ans : ix = 0.031 A
E.P 3.29
Find the current through 4 ohm resistor using Millman’s theorem.
www.allsyllabus.com

Figure E.P. 3.29

Ans : I = 2.05 A
E.P 3.30
Find the current through the impedance of (10 + j 10) Ω using Millman’s theorem.

Figure E.P. 3.30

Ans : 3.384 /12.6 A

vtu.allsyllabus.com
www.allsyllabus.com

276 j Network Theory

E.P 3.31
Using Millman’s theorem, find the current flowing through the impedance of (4 + j 3) Ω.

Figure E.P. 3.31

Ans : 3.64 / 15.23 A

www.allsyllabus.com

vtu.allsyllabus.com

You might also like